Sunteți pe pagina 1din 84

Q1.

What are the factors that distinguish Indus Valley Civilization (Harappan) from the
contemporary cultures in West Asia?
(1) The Indus Valley Civilization planned its cities with chessboard system, street, drainage
pipes, whereas the Mesopotamian cities showed a haphazard growth.
(2) The Harappan sites had rectangular houses with brick-lined bathrooms and wells, with
their stairways; such town-planning is not found in cities of Western Asia.
(3) The people of Western Asia showcased much better skills in the use of burnt bricks than
the Harappans.
Select the correct answer using the code given below:
a. 2 and 3 only
b. 1 and 3 only
c. 1 and 2 only
d. 1, 2 and 3

Answer: c
Explanation: The use of burnt bricks in the Harappan cities is remarkable because in the
contemporary buildings of Egypt dried bricks were mainly used. Even though the use of
baked bricks in contemporary Mesopotamia was found, but in Harappan cities, they were
used to a much larger extent. So statement 3 is not correct.
The drainage system in almost all cities, every big or small house had its courtyard and
bathroom. Drains were covered with bricks and sometimes with stone slabs. The street
drains were equipped with manholes. The drainage system of Harappa is almost unique,
and perhaps no other civilisation gave so much attention to health and cleanliness as the
Harappans. So statements 1 and 2 are correct.
Therefore the right option is c.

Q2. Consider the following statements:


(1) The Harappans did not use metallic money but possessed necessary raw materials for
all commodities they produced.
(2) The Harappans knew the use of wheel; however, carts with solid wheels were not in use
in Harappa.
Which of the statements given above is/are incorrect?
a. 1 only
b. 2 only
c. Both 1 and 2
d. Neither 1 nor 2

Answer: c
Explanation: Harappans did not use metallic money neither they possessed necessary raw
materials for commodities they produced. They carried on their exchanges through barter.
They procured metals from neighbouring areas by boats and bullock-carts, in return for
finished goods and foodgrains. So statement 1 is incorrect.
They had commercial links with Rajasthan, Afghanistan and Iran. Harappans knew the use
of wheel and carts with solid wheels were in use in Harappa. It also appears that Harappans
used some type of modern ekkas. So statement 2 is also incorrect.
Hence, the right option is c.

Q3. Which among the following statements about the Indus Valley Civilization is correct?
(1) In a terracotta figurine discovered at Mohenjodaro, a plant is shown growing out of the
embryo of a woman.
(2) The Harappans did not keep gods in the temple, a practice which was common in ancient
Egypt and Mesopotamia.
(3) The Harappan culture is poor in artistic works made of stone.
Select the correct answer using the code given below:
a. 3 only
b. 2 and 3 only
c. 1 and 3 only
d. 1, 2 and 3
Answer: b
Explanation: In Harappa, number of terracotta figurines of women have been found. In a
terracotta figurine discovered at Harappa, a plant is shown growing out of the embryo of a
woman. Probably the image represented goddess of earth and was intimately connected
with the origin and growth of plants. The Harappans looked upon earth as a fertility goddess
and worshipped her in the same manner as the Egyptians worshipped the Nile Goddess
Isis. So statement 1 is incorrect.
The inhabitants of Indus Valley Civilization worshipped gods in the form of trees, animals
and human beings. Most important animal worshipped was the humped bull, represented
on seals. But Harappans did not keep gods in temple, a practice which was common in
ancient Egypt and Mesopotamia. So statement 2 is correct. No massive work of art in stone
has been found as compared to the sculpture of ancient Egypt and Mesopotamia. So the
statement is true.
Therefore the right answer option is b.

Q4. Consider the following statements with regard to the advent of Aryans:
(1) The most important animal to play a part in Aryan life was the humped bull.
(2) The Rigveda, which is the earliest specimen of the Indo-European language, provides
information about the Aryans.
(3) The oldest text in the Iranian language, Avesta has similarities with the Rigveda of
Aryans.
Which of the statements given above is/are incorrect?
a. 2 and 3 only
b. 1 only
c. 1 and 2 only
d. 1, 2 and 3

Answer: b
Explanation: On their way to India, Aryans first appeared in Iran, here the Indo-Iranians
lived for a long time. The swiftness of horses enabled Aryans to make successful inroads
into Western Asia from 2000 BC onwards. Although the Aryans used several animals, the
horse played the most significant role in their life. Moreover, since they did not lead a settled
life, horses played a vital role in moving swiftly to new areas for pasturelands. Therefore
statement 1 is not correct.
Rigveda is the earliest specimen of the Indo-European language. Mitanni inscriptions of the
fourteenth century BC gives evidence that a branch of Aryans moves east from Iran. So
statement 2 is correct. Avesta and Rigveda use same names for several gods and even for
social classes. Thus statement 3 is correct. Therefore, the right option is b.

Q5. Which of the statements given below regarding the Aryans is correct?
a. The officer who enjoyed authority over the pasture grounds during Rigvedic times was
called Sangrihitri.
b. The Aryans possessed chariots driven by horses and introduced them into West Asia
and India.
c. Ploughshare is not mentioned in the Rigveda.
d. Land formed a well-established type of private property during the Rigvedic period and
was gifted to the priests.

Answer: b
Explanation: In the later Vedic period, collection of taxes and tributes was deposited with
an officer named sangrihitri. So statement a is not correct.
In Rigveda, Indra is called Purandhara, which means breaker of the fort. According to
Rigveda, Indra defeated many adversaries. The success of Aryans came because the
Aryans possessed chariots driven by horses and introduced them for the first time into West
Asia and India. Their soldiers were equipped with better arms. So statement b is correct.
Aryan people possessed a better knowledge of agriculture. Ploughshare is mentioned in
the earliest part of Rigveda. Possibly this ploughshare was made of wood. They were
acquainted with sowing, harvesting and thrashing. So statement c is not correct.
There are many references to the cow in the Rigveda which proves that Aryans were
pastoral people. Most of their wars were fought for the sake of cows. The term for war in
Rigveda is Gavisthi or search of cows. Cows and women slaves were given as gifts to
priests and never in terms of measurement of land. So statement d is not correct.
Therefore right option is b

Q6. Which among the following statements about the rise of Jainism and Buddhism is
correct?
a. Kaivalya is a term used for perfect knowledge in Buddhism.
b. Vardhamana Mahavira and Gautam Buddha disputed the authority of the Brahmanas.
c. Jainism did not recognise the existence of gods.
d. Jainism and Buddhism, both condemned the varna system

Answer: b
Explanation: Mahavira attained perfect knowledge of Kaivalya, at the age of 42. Through
Kaivalya, he conquered misery and happiness. Because of this conquest, he is called
Mahavira or jaina, i.e. the conqueror and his followers are called Jainas. So statement a is
not correct. Jainas recognized the existence of the gods but placed them lower than the
jina. So statement c is not correct.
Jainas did not condemn the varna system as Buddhism did. According to Mahavira, a
person is born in a high or in lower varna in consequence of the sins of the virtues acquired
by him in the previous birth. In his opinion, through pure and meritorious life members of
the lower castes can attain liberation. So statement d is not correct. Buddhism and Jainism
were against Brahmanic domination and therefore disputed the authority of Brahmanas. So
statement b is correct.

Q7. Which among the following are special features of Buddhism and the causes of its
spread?
(1) In comparison to Brahmanism, Buddhism was liberal and democratic.
(2) All rulers of Magadha embraced Buddhism, which led to its rapid spread outside India
into South-east Asia, China etc.
Select the correct answer using the code given below:
a. 1 only
b. 2 only
c. Both 1 and 2
d. Neither 1 nor 2

Answer: a
Explanation: Buddhism does not recognise the existence of god and soul, which was a
revolution in the history of Indian religions. It won the support of the lower orders as it
attacked the varna system. Women were also admitted to the sangha and thus brought on
par with men. So statement 1 is correct.
Embracing of Buddhism by Mauryan ruler Ashoka was an epoch-making event as he sent
missions to Central Asia, West Asia and Sri Lanka and thus transformed it into a world
religion. But it has to be noticed that not all rulers of Mauryan dynasty were Buddhists.
Chandragupta Maurya, the founder of the Mauryan empire, embraced Jainism. So
statement 2 is not correct.
Hence, the right option is a.

Q8. Consider the following statements with regard to the rise of the Magadhan empire:
(1) The rich iron deposits were situated nearby Magadha.
(2) Their capitals, Rajgir and Pataliputra, were situated at strategic points.
(3) Magadha lay at the centre of the middle Gangetic plain where fertile alluvium was
available
(4) Rulers of Magadha employed only fair and peaceful means at their disposal to enlarge
their kingdoms.
Which of the statements given above is/are incorrect?
a. 1 and 2 only
b. 2 and 3 only
c. 1 and 4 only
d. 4 only
Answer: d
Explanation: The formation of the Magadhan state was work of several ambitious rulers
such as Bimbisara, Ajatashtru and Mahapadma Nanda, who employed all means, fair and
foul at their disposal to enlarge their kingdoms and to strengthen their states. So statement
4 is not correct.
Magadha was located on the upper and lower parts of the Gangetic valley. It was located
on the mainland route between west and east India. Both Rajgir and Pataliputra were
located in strategic positions. So statement 2 is correct.
The area had fertile soil. It also received enough rainfall. Magadha was encircled by rivers
on three sides, the Ganga, Son and Champa making the region impregnable to enemies.
So statement 3 is correct.
Magadha had huge copper and iron deposits. Because of its location, it could easily control
trade. Availability of iron enabled them to develop advanced weaponry. They were also the
first kings to use elephants in the army. So statement 1 is correct.
Therefore the right answer option is d.

Q9. Consider the following statements with regard to the Mauryan Empire:
(1) Ashoka was the first Indian king to speak directly to the people through his inscriptions.
(2) Rajukas were appointed by Chandragupta Maurya who rewarded and punished people.
Which of the statements given above is/are incorrect?
a. 1 only
b. 2 only
c. Both 1 and 2
d. Neither 1 nor 2

Answer: b
Explanation: Within the Mauryan empire, it was not Chandragupta Maurya, but Ashoka
who appointed a class of officers known as the Rajukas. These officers were vested with
the authority of not only rewarding people but also punishing them, wherever necessary.
Therefore statement 2 is incorrect.
Ashoka’s inscriptions were engraved on rocks, polished stone pillars mounted by capitals
and in the caves. They are found in Indian subcontinent and Kandahar in Afghanistan.
These inscriptions are in the form of 44 royal orders and each royal order has several
copies. So statement 1 is correct.
Therefore the right answer option is b.

Q10. Which of the statements given below is incorrect?


a. For the first time in the Maurya period, slaves were engaged in the agricultural work on
a large scale.
b. Ancient Indian society was a slave society like Greece and Rome.
c. According to Megasthanes, he did not notice any slaves in India
d. All of the above

Answer: b
Explanation: Ancient Indian society was not a slave society. What slaves did in Greece
and Rome was done by Shudras in India. Shudras were compelled to serve the three higher
varnas as slaves, artisans, agricultural labourers and domestic servants. Therefore
statement b is incorrect.
The Mauryan state maintained farms on which numerous slaves and hired labourers were
employed. So statement a is correct. Domestic slaves were found in India from Vedic
times onwards, even though Megasthenes states that he did not notice any slaves in India.
So statement c is correct.

Q11. Consider the following statements with regard to the Mauryan economy:
(1) Punch-marked coins which carry the symbols of the peacock and the hill and crescent
formed the imperial currency of the Mauryas.
(2) The enormous expenditure on the army and payment to bureaucracy created a financial
crisis for the Mauryan empire later.
(3) Samaharta was the chief custodian of the state treasury and store-house.
Which of the statements given above is/are incorrect?
a. 1 only
b. 2 and 3 only
c. 3 only
d. 1 and 2 only

Answer: c
Explanation:
Statement 3 is incorrect as Samaharta in the Mauryan administration was the highest
officer-in-charge of assessment, and sannidhata was chief custodian of the state treasury
and store-house. Mauryas attached greater important to assessment than to storage and
depositing.
Statement 2 was one of the financial causes of the downfall of the Mauryan Empire. Despite
all kinds of taxes imposed on the people, it was difficult to maintain the large army and large
regiment of the officer. So statement 2 is correct.
Ashoka also made large grants to the Buddhist monks which left the royal treasury empty.
Brahmanical reaction, oppressive rule at provinces, neglect of North-West Frontier were
some of the other reasons of downfall of the empire.

Q12. Consider the following statements with regard to the Pandya Kingdom:
(1) The Pandya Kingdom, according to Megasthenes, was known for pearls.
(2) The Pandya society was patriarchal.
(3) The chief centre of Pandya political power was at Uraiyur, which was known for the
cotton trade.
Which of the statements given above is/are correct?
a. 1 only
b. 2 and 3 only
c. 2 only
d. 1 and 3 only
Answer: a
Explanation: As per Megasthenes, the Pandya kingdom was ruled by a woman, which
suggests that the Pandya society was matriarchal and not patriarchal. Therefore statement
2 is incorrect.
The Pandya kingdom had its capital at Madurai and was wealthy and prosperous. The
Pandya kings profited from trade with the Roman empire and sent embassies to the Roman
emperor Augustus. The chief centre of the political power of Cholas lay at Uraiyur which
was famous for cotton trade and for the Pandya it was Madurai. Therefore, statement 3 is
incorrect.
Therefore option a is the only correct option.

Q13. Which of the statements with regard to art and craft in post-Maurya age is correct?
(1) A special type of cloth called ‘Sataka’ was manufactured at Mathura.
(2) All Kushan gold coins were minted out of Roman gold.
Select the correct answer using the code given below:
a. 1 only
b. 2 only
c. Both 1 and 2
d. Neither 1 nor 2

Answer: a
Explanation: In north India, Indo-Greek rulers issued a few gold coins. But the Kushans
issued gold coins in large numbers. However, it is wrong to think that all Kushan gold coins
were minted out of Roman gold. As early as fifth century BC, India had paid a tribute of 320
talents of gold to the Iranian empire. This gold may have been extracted from gold mines in
Sindh. The Kushans probably obtained gold from Central Asia. They may also have
procured it either from Karnataka or from gold mines of Dhalbhum in South Bihar which
later came under their rule. Therefore, statement 2 is incorrect.
Statement 1 is correct as cloth-making, silk weaving made great progress during Post
Maurya period. Therefore option a is the right option.

Q14. Consider the following statements with regard to the Gupta Empire:
(1) During the Gupta period, priestly landlords were given a land grant.
(2) Fa-Hien visited India during the reign of Samudragupta.
(3) India’s trade with the Roman Empire was adversely affected when the art of growing silk
was learned by the Romans.
Which of the statements given above is/are incorrect?
a. 2 only
b. 2 and 3 only
c. 1 and 2 only
d. 1 only

Answer: a
Explanation: Samudragupta was called as Napolean of India by historians. However, it
was in Chandragupta II’s time that Chinese pilgrim Fa-Hien (399-414) visited India and
wrote an elaborate account of the life of its people. Therefore statement 2 is incorrect.
The growing practice of land grants to priests, for religious and other purposes, reduced
revenues of Gupta empire in the long run. Attacks of Hunas and dwindling foreign trade with
the Roman empire further hastened the decline of the Gupta empire. Both statements 1
and 3 are correct.
So option a is the correct option.

Q15. Which of the statements given below is correct?


a. Land taxes decreased in number during the Gupta period; however, those on trade and
commerce increased.
b. In the Gupta empire, uparika was in-charge of vishayas.
c. Buddhism received royal patronage during the Gupta period as in the days of Asoka and
Kanishka.
d. Kalidas wrote Abhijanasakuntalam and Amarkosa was written by Amarsimha.

Answer: d
Explanation: Kalidasa and Amarsimha were luminaries in the court of Chandragupta II. So
statement d is correct. In the Gupta period, land taxes increased in number and those on
trade and commerce decreased. Kings collected taxes varying from one-fourth to one-sixth
of the produce. Apart from that, peasants had to supply foodgrains, animals when the royal
army passed. In some parts of the country, forced labour was also practised. So statement
a is incorrect.
Gupta empire was divided into bhuktis (divisions), vishayas (districts), vithis and villages.
Each bhukti was under the charge of an uparika. The bhuktis were divided into districts
(vishayas) which were placed under charge of vishayapati. Vishayas were divided into vithis
and which were again divided into villages. Therefore statement b is incorrect.
Buddhism no longer received royal patronage in the Gupta period, although Fa-Hien gives
the impression that this religion was very flourishing. Especially it suffered in Magadha.
Brahmanism had gained prominence, Vishnu and Siva commanded large followership. So
statement c is not correct.
Hence, the correct option is d.

Q16. Which of the statements given below is incorrect?


a. Banabhatta wrote a book called Harshacharita.
b. Harsha's administration was more feudal and decentralised than that of the Guptas.
c. Hiuen Tsang visited India during Harsha's reign.
d. Nalanda was the most famous centre of Hinayana Buddhism during Harsha's age.

Answer: d
Explanation: Most famous centre of Buddhism during Harsha's age was Nalanda, which
taught Buddhist philosophy of Mahayana. Therefore, statement d is incorrect.
The early history of Harsha’s rule is reconstructed from a study of Banabhatta, who was his
court poet and who wrote a book called Harshacharita. This can be supplemented by the
account of the Chinese pilgrim Hsuan Tsang who visited India in the seventh century AD
and stayed in the country for about 15 years. So statement a and c are correct.
Harsha continued land grants to priests and gave land grants to officers also, which further
made his empire more feudal and decentralised. So statement b is correct.

Q17. Consider the following statements:


(1) Sanskrit was the official language in the peninsula since the Mauryan period.
(2) The Aihole inscription was issued in the Sanskrit language by Ravikirti, the court poet of
Pulakesin II
Which of the statements given above is/are correct?
a. 1 only
b. 2 only
c. Both 1 and 2
d. Neither 1 nor 2

Answer: b
Explanation: Epigraphs between 2nd century BC and third century AD were mostly written
in Prakrit. The Brahmi inscriptions found in Tamil Nadu also contain Prakrit words. From
about 400 AD, Sanskrit became the official language in the peninsula and most charters
were composed in it. So statement 1 is incorrect.
Aihole inscription is an example of poetic excellence reached in Sanskrit language, it was
issued by Ravikirti, the court poet of Pulakesin II. So statement 2 is correct.
Therefore option b is the right option.

Q18. Consider the following statements with regard to India’s contact with Asian countries:
(1) The name Suvarnabhumi was given to Pegu and Moulmein in Burma and Suvarnadwipa
to Java in Indonesia by ancient Indians.
(2) The Cholas were the first to found their colonies in Sumatra, which later flourished into
the kingdom of Sri Vijaya.
(3) The rulers of Kamboja and Champa were devotees of Buddha and developed their
kingdom as the centres of Sanskrit learning.
Which of the statements given above is/are correct?
a. 1 and 2 only
b. 2 and 3 only
c. 1 and 2 only
d. 1 only

Answer: d
Explanation: The Indian culture spread to South-east Asia, but not through the medium of
Buddhism. Except in the case of Burma, it was mostly diffused through the Brahmanical
cults. Pallavas founded their colonies in Sumatra, which later flourished into the kingdom of
Sri Vijaya. So statement 2 is not correct.
In Indo-China, Indians set up two powerful kingdoms in Kamboja (Cambodia) and Champa
(Vietnam). Ruler of Kamboja was a devotee of Siva and developed kingdom as a centre of
Sanskrit learning. The King of Champa was a Saiva and Sanksit was its official language.
So statement 3 is not correct.
Statement 1 is correct, so the right option is d.

Q19. With reference to the beads found in the Indus Valley Civilisation, consider the
following statements:
(1) The beads were made of steatite and lapis lazuli.
(2) The factories of beads were found at Rakhigarhi and Lothal.
Which of the statements given above is/are correct?
a. 1 only
b. 2 only
c. Both 1 and 2
d. Neither 1 nor 2

Answer: a
Explanation: Hoards of jewellery found at Mohenjodaro and Lothal include necklaces of
gold and semi-precious stones, copper bracelets and beads, gold earrings and head
ornaments, faience pendants and buttons, and beads of steatite and gemstones. All
ornaments are well crafted.
Beads were made of cornelian, amethyst, jasper, crystal, quartz, steatite, turquoise, lapis
lazuli, etc. Metals like copper, bronze and gold, and shell, faience and terracotta or burnt
clay were also used for manufacturing beads. The beads are found in varying shapes and
sizes. Hence, statement 1 is correct
The bead industry seems to have been well developed as evident from the factories
discovered at Chanhudaro and Lothal. So, Statement 2 is incorrect.

Q20. Consider the following pairs:

1. Bronze Bull : Kalibangan


2. Dancing Girl : Mohen-jo-daro
3. Copper Dog : Chanhudaro
4. The Bearded Priest : Harappa

Which of the pairs given above is/are correctly matched?


a. 1 and 2 only
b. 2 and 4 only
c. 1, 2 and 4 only
d. 1, 2, 3 and 4

Answer: a
Explanation: Some of the essential findings of the Indus Valley Civilisation is as follows:
The Bronze Bull: Kalibangan
The copper dog and bird: Lothal
The Dancing Girl and the Bearded Priest: Mohenjo-Daro.
The Pashupati Seal: Mohenjo-Daro
The Mother Goddess and Bull Statues: Kot Diji
Extra Information: The art of bronze-casting was practised on a wide scale by the
Harappans. Their bronze statues were made using the ‘lost wax’ technique in which the
wax figures were first covered with a coating of clay and allowed to dry. Then the wax was
heated, and the molten wax was drained out through a tiny hole made in the clay cover.
The hollow mould thus created was filled with molten metal which took the original shape
of the object. Once the metal cooled, the clay cover was completely removed.

Q21. With reference to the Buddhist stupas, consider the following statements:
(1) Events from the life of the Buddha, the Jataka stories, were depicted on the railings and
gates (torans) of the stupas.
(2) The Sanchi stupa is a UNESCO World Heritage Site and was built during the Gupta
period.
Which of the statements given above is/are correct?
a. 1 only
b. 2 only
c. Both 1 and 2
d. Neither 1 nor 2

Answer: a
Explanation:
A stupa consists of the following parts:
The stupa consists of a hemispherical dome (anda)
A harmika and chhatra on the top.
The circumambulatory path (pradakshinapatha), with railings and sculptural decoration
Torans were ceremonial gateways around the stupas.
The events from the life of the Buddha, the Jataka stories, were depicted on the railings
and torans of the stupas. Mainly synoptic narrative, continuous narrative and episodic
narrative are used in the pictorial tradition. The main events associated with the Buddha’s
life which were frequently depicted were events related to the birth, renunciation,
enlightenment, Dhammachakrapravartana, and Mahaparinibbana (death). So, Statement
1 is correct.
Some Important stupas in India are found in Sanchi, Sarnath, Amravati and Bharhut.
The Sanchi Stupa was built by Emperor Ashoka (Maurya Dynasty) in the 3rd Century BCE
and was declared a UNESCO World Heritage Site in 1989. (Not during Gupta period). So,
Statement 2 is incorrect.

Q22. With reference to the famous Ajanta Caves, consider the following statements:
(1) It is located in Maharashtra and has 22 caves only.
(2) The walls of the caves are decorated with images of Buddha and Bodhisattvas, the
largest being the Mahaparinirvana image.
Which of the statements given above is/are correct?
a. 1 only
b. 2 only
c. Both 1 and 2
d. Neither 1 nor 2

Answer: b
Explanation: The Ajanta Caves is located in Aurangabad District of Maharashtra State and
has twenty-nine caves and four Chaitya caves. So, Statement 1 is incorrect.
It has large chaitya viharas and is decorated with sculptures and paintings. Ajanta is the
only surviving example of painting of the first century BCE and the fifth century CE. The
caves at Ajanta as well as in western Deccan, in general, have no precise chronology
because of the lack of known dated inscriptions. Their facade is decorated with Buddha
and, Bodhisattva images, the biggest one being the Mahaparinibbana image. So,
Statement 2 is correct.

Q23. Consider the following pairs:

King Temple
1 Narshimvarman II : Shore Temple
2 Rajaraja Chola : Brihadeshwara
Temple
3 Kulottunga III : Gangaikonda
Cholapuram

Which of the pairs given above are correctly matched?


a. 1 and 2 only
b. 2 and 3 only
c. 3 only
d. 1, 2 and 3

Answer: a
Explanation: The Shore Temple at Mahabalipuram was built during the reign of
Narshimhavarman II.
In the eleventh century, when the Cholas reached their zenith of power, the masterpieces
of Chola art and architecture began to appear. The temples of Brihadeswara at Thanjavur,
Gangaikonda Cholapuram and Darasuram were built during the reigns of Rajaraja Chola
and his son, Rajendra Chola. Hence Option 1 and 2 are correct.
Kulothunga Chola III commissioned the Sarabeswara or Kampahareswara temple at
Tribhuvanam near Kumbakonam which is considered a great specimen of Dravidian
Architecture. So, option 3 is incorrect.
Some of the key features of the Chola temples:
Dravidian Style of architecture
Principal deity: Lord Shiva and was placed in the garbagriha.
Use of miniature Gods and goddesses and dvarapalas.

Q24. With reference to the Nagara style of temple architecture, consider the following
statements:
(1) It was a style of temple architecture that was popular in the northern part of India.
(2) It is characterised by elaborate gateways or gopurams.
(3) There are various subtypes of Nagara style architecture like the Latina, Phamasana and
the Vallabhi type.
Which of the statements given above are correct?
a. 1 and 2 only
b. 2 and 3 only
c. 1 and 3 only
d. 1, 2 and 3

Answer: c
Explanation: Nagara style of temple architecture is popular in northern India. So,
Statement 1 is correct.
Unlike Dravida Style (in South India) it does not usually have elaborate boundary walls or
gateways. Hence, Statement 2 is incorrect.
There are many subdivisions of Nagara temples depending on the shape of the Shikhara.
The simple shikhara which is square at the base and whose walls curve or slope inward to
a point on top is called the 'latina' or the rekha-prasada type of shikhara.
The second major type of architectural form in the nagara order is the phamsana.
Phamsana buildings tend to be broader and shorter than latina ones.
The third main sub-type of the nagara building is what is generally called the valabhi type.
These are rectangular buildings with a roof that rises into a vaulted chamber.
Therefore, statement 3 is correct.

Q25. With reference to the Nalanda University, consider the following statements:
(1) Skandagupta founded it.
(2) Buddhist doctrines only related to Mahayana were taught here.
(3) The sculptures at Nalanda were heavily inspired by the Buddhist art at Sanchi and drew
inspiration from it.
Which of the statements given above are incorrect?
a. 1 and 2 only
b. 2 and 3 only
c. 1 and 3 only
d. 1, 2 and 3

Answer: d
Explanation: Most of the information about Nalanda is based on the records of Xuan
Zang—previously spelt as ‘Hsuan-tsang’— which states that the foundation of a monastery
was laid by Kumargupta I in the fifth century CE. So, Statement 1 Is Incorrect.
There is evidence that all three Buddhist doctrines— Theravada, Mahayana and Vajrayana
were taught here and monks made their way to Nalanda and its neighbouring sites of Bodh
Gaya and Kurkihar from China, Tibet and Central Asia in the north. The Nalanda sculptures
initially depict Buddhist deities of the Mahayana sect. So, Statement 2 is incorrect.
The sculptural art of Nalanda, in stucco, stone and bronze, developed out of heavy
dependence on the Buddhist Gupta art of Sarnath. By the ninth century, a synthesis
occurred between the Sarnath Gupta idiom, the local Bihar tradition, and that of
central India, leading to the formation of the Nalanda School of sculpture characterised
by distinctive facial features, body forms and treatment of clothing and jewellery. Thus,
Statement 3 is incorrect.

Q26. Which dance form had greatly inspired the painters of the Kerala Mural?
a. Kathakali
b. Mohiniattam
c. Kuchipudi
d. Bharatnatyam

Answer: a
Explanation: Kerala Murals - Kerala painters (during the period from the sixteenth to the
eighteenth century) evolved a pictorial language and technique of their own while
discriminately adopting certain stylistic elements from Nayaka and Vijayanagara schools.
The painters evolved a language taking cues from contemporary traditions like
Kathakali and kalam ezhuthu using vibrant and luminous colours, representing
human figures in three-dimensionality. Most of the paintings are seen on the walls of
shrines and cloister walls of temples and some inside palaces. Thematically too, paintings
from Kerala stand apart. Most of the narrations are based on those episodes from Hindu
mythology which were popular in Kerala. The artist seems to have derived sources from
oral traditions and local versions of the Ramayana and the Mahabharata for painted
narration.
More than sixty sites have been found with mural paintings which include three palaces—
Dutch palace in Kochi, Krishnapuram palace in Kayamkulam and Padmanabhapuram
Palace. Among the sites where one can see the mature phase of Kerala’s mural painting
tradition are Pundareekapuram Krishna temple, Panayanarkavu, Thirukodithanam,
Triprayar Sri Rama temple and Trissur Vadakkunathan temple.

Q27. With reference to the Udaigiri-Khandagiri caves of Odisha, consider the following
statements:
(1) They have inscriptions of Pallava kings.
(2) The caves were made for Buddhist monks.
Which of the statements given above is/are incorrect?
a. 1 only
b. 2 only
c. Both 1 and 2
d. Neither 1 nor 2

Answer: d
Explanation: The rock-cut cave tradition also existed in Odisha. The earliest examples are
the Udaigiri-Khandagiri caves in the vicinity of Bhubaneswar. These caves are scattered
and have inscriptions of Kharavela kings. According to the inscriptions, the caves were
meant for Jain monks. There are numerous single-cell excavations. Some have been
carved in huge independent boulders and given the shape of animals. Hence, Both
Statement 1 and 2 are incorrect.

Q28. Consider the statements with reference to chaityas and viharas:


(1) These were initially concerned with the wooden architecture.
(2) The chaityas are the dwelling place of the monks while the viharas are the prayer halls.
Which of the statements given above is/are incorrect?
a. 1 only
b. 2 only
c. Both 1 and 2
d. Neither 1 nor 2

Answer: b
Explanation: Initially the Chaitya and Vihara architecture was concerned with the
wooden architecture, but over time, the rock-cut caves came into prominence. So,
Statement 1 is correct.
Chaityas are the prayer halls while Viharas are the dwelling place for the monks. So,
Statement 2 is incorrect.
Extra Information:
A square mandapa was constructed inside the cave, which was surrounded by the dwelling
places for the monks. The Chaityas belonging to 200 BC-200 AD are mainly related to
Hinayana Buddhism. Bhaja, Kondane, Pitalkhora, Ajanta (9th – 10th caves), Beda, Nasik
and Karle caves are the examples such architecture. There is a lack of image sculpture and
these caves mostly contain the simple stupas.

Q29. Consider the following statements:


(1) Both Buddhism and Jainism were part of the shraman tradition.
(2) Yaksha worship was assimilated in Buddhism but not in Jainism.
Which of the statements given above is/are correct?
a. 1 only
b. 2 only
c. Both 1 and 2
d. Neither 1 nor 2

Answer: a
Explanation: Shramana was an ancient Indian religious movement with origins in the Vedic
religion. However, it took a divergent path, rejecting Vedic Hindu ritualism and the authority
of the Brahmins—the traditional priests of the Hindu religion. The Sramana movement
gave rise to Jainism and Buddhism. So, Statement 1 is correct.
The worship of Yakshas and mother goddesses were prevalent before and after the advent
of Buddhism, it was assimilated in Buddhism and Jainism. So, multiple forms of worship
existed. Nevertheless, Buddhism became the most popular social and religious movement.
Hence, Statement 2 is incorrect.

Q30. Consider the following statements about seals of the Indus Valley Civilization:
(1) These seals are engraved in the pictographic script which has not been deciphered yet.
(2) Images of cow and elephant have been depicted in the Pashupati seal found in Mohenjo-
daro.
Which of the statements given above is/are correct?
a. 1 only
b. 2 only
c. Both 1 and 2
d. Neither 1 nor 2

Answer: a
Explanation: Archaeologists have discovered thousands of seals usually made of steatite,
faience and terracotta with beautiful figures of animals like the unicorn, tiger, goat, buffalo,
elephant etc. Every seal is engraved in a pictographic script which is yet to be deciphered.
So, Statement 1 is correct.
The most remarkable seal is the one depicted with a figure in the centre and animals around.
This seal is generally identified as the Pashupati Seal by some scholars. This seal depicts
a human figure seated cross-legged. An elephant and a tiger are depicted to the right side
of the seated figure, while on the left a rhinoceros and a buffalo are seen (Figure of the
cow was not there). In addition to these animals, two antelopes are shown below the seat.
So, Statement 2 is incorrect.

Q31. Consider the following statements:


(1) The Ellora caves are only related to Buddhism.
(2) The Elephanta Caves are contemporary of the Ellora caves.
Which of the statements given above is/are correct?
a. 1 only
b. 2 only
c. Both 1 and 2
d. Neither 1 nor 2

Answer: b
Explanation: An important cave site located in Aurangabad District is Ellora. It is located a
hundred kilometres from Ajanta. It is a unique art-historical site in the country as it has
monasteries associated with the three religions (Hinduism, Buddhism and Jainism) dating
from the fifth century CE onwards to the eleventh century CE. So, Statement 1 is incorrect.
The Elephanta Caves located near Mumbai were originally a Buddhist site which was later
dominated by the Shaivite faith. It is contemporary with Ellora, and its sculptures show
slenderness in the body, with stark light and dark effects. Hence, Statement 2 is correct.

Q32. Which among the following sites emerged as important centres of art production from
the first century CE onwards?
(1) Gandhara
(2) Mathura
(3) Vengi
Select the correct answer using the code given below:
a. 1 and 2 only
b. 2 and 3 only
c. 1 and 3 only
d. 1, 2 and 3

Answer: d
Explanation: The first century CE onwards, Gandhara ( Pakistan), Mathura (North India)
and Vengi in Andhra Pradesh emerged as important centres of art production.
Buddha in the symbolic form got a human form in Mathura and Gandhara. The sculptural
tradition in Gandhara had the confluence of Bactria, Parthia and the local Gandhara
tradition. The local sculptural tradition at Mathura became so strong that the tradition spread
to other parts of northern India. The best example in this regard is the Stupa sculptures
found at Sanghol in Punjab.
Vengi is a town in present-day Andhra Pradesh, and one of the places where Amravati
School of Art flourished. It is characterised by intense emotions as the figures are slim, have
a lot of movement, and bodies are shown with three bents (i.e. Tribhanga). Therefore, all
options are correct.

Q33. Which of the following statements is not correct about the Government of India Act
1935?
a. The Act provided for the establishment of an All-India Federation based on the Union of
the British Indian provinces only.
b. The provinces were to be governed under a new system based on provincial autonomy
under which elected ministers controlled all provincial departments.
c. The Governor could veto legislative and administrative measures related to British
business interests
d. Defence and foreign affairs remained outside the control of the federal legislature.

Answer: a
Explanation: According to this Act, India would become a federation if 50% of Indian states
decided to join it. They would then have a large number of representatives in the two houses
of the central legislature. However, the provisions with regards to the federation were not
implemented. The act made no reference even to granting dominion status, much less
independence, to India. The act provided for the establishment of a “Federation of India” to
be made up of both British India and some or all of the princely states. So statement a is
incorrect.
Concept of provincial autonomy was given in place of dyarchy, as given in GOI act, 1919.
So statement b is correct.
But still elected representatives could be checked by the Governor, who can use veto power
in legislation and administration for safeguarding British business interests. So statement
c is correct. Dyarchy was introduced at the central level, where defence and foreign affairs
remained outside the control of the legislature. So statement d is correct.

Q34. Consider the following statements in regard to the role of the Princely States during
the Indian National Movement:
(1) The first All India States’ People’s Conference was held in 1927.
(2) The princes were given two-fifth seats in the Upper House and one-third of the seats in
the Lower House, in the proposed federal structure of the Government of India Act, 1935.
(3) With the launching of the Civil Disobedience Movement, Congress made no distinction
between British India and the Princely States and the call for the struggle was extended to
the people of the Princely States.
Which of the statements given above are correct?
a. 1 and 2 only
b. 2 and 3 only
c. 1 and 3 only
d. 1, 2 and 3

Answer: a
Explanation: With the launching of the Quit India Movement and not the Civil Disobedience
Movement, the Congress made no distinction between British India and the Indian States
and call for the struggle was extended to the people of the states. So statement 3 is
incorrect. The first conference of All India States’ People’s Conference was held in 1927 to
coordinate political activities in the different states. So statement 1 is correct.
British used princes to prevent the growth of national unity. In the Government of India Act,
1935, the federal structure was so planned as to check forces of nationalism. Princes were
given two-fifth of seats in Upper house and one-third of the seats in the Lower House, in
the proposed federal structure of Government of India Act, 1935. So statement 2 is
correct.
Hence the correct answer is option a.

Q35. According to the Doctrine of Lapse, when the ruler of a protected state died without a
natural heir, his state was not to pass to an adopted heir as sanctioned by the age-old
tradition of the country. Following states were annexed by applying this doctrine:
(1) Nagpur
(2) Satara
(3) Sambalpur
(4) Balaghat
(5) Jhansi
Which of the following is the correct chronological order of the annexation of the states?
a. 2-3-4-5-1
b. 2-3-1-5-4
c. 3-1-2-4-5
d. 3-1-4-5-2

Answer: a
Explanation: Nagpur was annexed in 1854, Satara in 1848, Sambalpur in 1849, Balaghat
in 1850, and Jhansi in 1853 under the doctrine of lapse. The Doctrine of Lapse was an
annexation policy followed widely by Lord Dalhousie when he was India’s Governor-General
from 1848 to 1856. According to this, any princely state under the direct or indirect (as a
vassal) control of the East India Company where the ruler did not have a legal male heir
would be annexed by the company. Nana Sahib and the Rani of Jhansi had grievances
against the British because the former’s pension was stopped by the British after his foster
father died, and the Rani’s adopted son was denied the throne under the doctrine of lapse.

Q36. Which of the following statements is incorrect?


a. The Charter Act of 1813 directed the East India Company to spend the sum of one lakh
of rupees, for the encouragement of modern sciences and the sum was made available by
the Company immediately.
b. The Government of India decided in 1835 to impart education in schools and colleges
with English as the medium of instruction.
c. The Wood’s Dispatch asked the Government of India to assume the responsibility for the
education of masses.
d. The Government of India devoted only about one crore of rupees to education out of its
total net revenue of nearly 47 crores till 1886.

Answer: a
Explanation: The British Parliament renewed the charter of the East India Company in
1813, it had required the Company to spend 100,000 rupees per year for the introduction
and promotion of knowledge of the modern sciences. However, it was only under the
English Education Act of 1835 that put into effect the decision to reallocate the funds to the
East India Company to spend on education and literature in India. So statement a is
incorrect and statement b is correct.
Charles Wood, the President of the Board of Control of the English East India Company,
had a great impact on disseminating education in India when in 1854 he sent a despatch to
Lord Dalhousie, the then Governor-General of India and suggested that Primary Schools
must adopt vernacular languages. He also suggested that high schools use Anglo-
Vernacular Medium and that English should be the medium for college-level education. So
statement c is correct.
The Government of India devoted only about one crore of rupees to education out of its
total net revenue of nearly 47 crores till 1886. The government was never willing to spend
more than a scanty sum on education. So statement d is correct

Q37. Consider the following statements in regard to Sayyid Ahmad Khan:


(1) He was very much impressed by modern scientific thoughts and worked all his life to
reconcile it with Islam.
(2) Towards the end of his life, he encouraged his followers to join the rising national
movement.
(3) Primarily, he was not a communalist.
Which of the statements given above are correct?
a. 1 and 2 only
b. 2 and 3 only
c. 1 and 3 only
d. 1, 2 and 3

Answer: c
Explanation: Sayyid Ahmad Khan, towards the end of his life, discouraged his followers to
join the rising national movement. He only wanted the backwardness of the Muslim middle
and upper class to go. So statement 2 is incorrect. He was basically not a communalist.
He supported Dadabhai Naoroji and Surendranath Banerjee in obtaining representation for
Indians in the government and the civil services. So statement 3 is correct. He believed
that Muslim society could move ahead only if rigid orthodoxy was abandoned and
pragmatism was adopted. So statement 1 is correct.

Q38. Consider the following statements with regard to ‘Permanent Settlement system’ of
land revenue:
(1) The work related to this system though started by Warren Hastings was completed by
Lord Cornwallis.
(2) Lord Cornwallis was assisted by Sir John Shore in adopting this settlement policy.
(3) Under this settlement, Zamindars were recognized as the owners of the land.
(4) Lord Cornwallis had adopted this settlement to teach a lesson to the peasants of India.
Which of the statements above given is/are incorrect?
a. 1 only
b. 2 and 3 only
c. 4 only
d. 2 and 4 only

Answer: c
Explanation: For the first time, Warren Hastings tried to introduce the land settlement in
1772 but in 1776, he adopted annual settlement. Lord Cornwallis started permanent
settlement system of land revenue in 1790. In this sense, Lord Cornwallis is said to have
completed the work of Warren Hastings. Sir John Shore assisted Lord Cornwallis in
adopting this land settlement. Under this settlement, Zamindars were recognized as the
owners of the land. They could remain as such so long they were paying rent prescribed.
Lord Cornwallis did not intend to teach a lesson to the peasants with this system but wanted
to realize the rent. Therefore statement 4 is incorrect, rest of the statements are
correct.
So right option is c

Q39. Which of the following statements is incorrect?


a. The first session of the Indian National Congress was presided over by W.C. Banerjee in
December 1885 and attended by 72 delegates.
b. Surendranath Banerjea was one of the founder members of the Indian National
Congress.
c. Dadabhai Naoroji organised the East India Association in London to discuss the Indian
matters and to influence British people to promote Indian welfare.
d. Pherozeshah Mehta, K.T. Telang, Badruddin Tyabji and others formed the Bombay
Presidency Association in 1885.
Answer: b
Explanation: Surendranath Banerjea and many other leaders of Bengal had not attended
the first session of the National Congress as they were busy with the Second National
Conference at Calcutta.
INC met in Calcutta in December 1886 under the president-ship of Dadabhai Naoroji.
Surendranath Banerjee founded the Indian National Association. He was elected the
Congress President in 1895 at Poona and in 1902 at Ahmedabad. So statement b is not
correct.
The first session of the Indian National Congress was held at Gokul Singh Tejpal Sanskrit
School, Bombay in December 1885. It was presided over by W.C. Banerjee and attended
by 72 delegates. So statement a is correct.
Dada Bhai Naoroji, who is known as Grand Old Man of India, organised the East India
association in 1866 in London to discuss the Indian matters and to influence British people
to promote Indian Welfare. So statement c is correct.
The first Muslim president of the Indian National Congress party was Badruddin Tyabji, who
along with Pherozeshah Mehta, K.T. Telang, and others formed the Bombay presidency
association in 1885. So statement d is correct.

Q40. Which of the following were aims and objectives of the Indian National Congress at
the time of its establishment in 1885?
(1) Founding a democratic and nationalist movement
(2) The political education of the people
(3) Framing Constitution for the Indian National Congress
(4) Promoting and nurturing Indian nationhood
Select the correct answer using the code given below:
a. 1, 2 and 3 only
b. 1, 2 and 4 only
c. 1, 3 and 4 only
d. 2, 3 and 4 only
Answer: b
Explanation: The Indian National Congress had the following aims and objectives when it
was established:
Inculcate a feeling of national unity and try to eradicate the notion of race, creed and
provincial prejudices.
Seek the co-operation of all the Indians in its efforts and allow them to take part in the
administrative affairs of the country.
Find a solution to the social problems of the country.
Real Aim of Congress was to train the Indian youth in political agitation and to organise or
to create public opinion in the country. For this, they used the method of an annual session
where they discussed the problem and passed the resolution.

Q41. Consider the following statements in respect of the Theosophical Society Movement
in India:
(1) The Theosophical Society was officially formed in New York City, United States, on 17th
November 1875 by Helena Petrovna Blavatsky, Colonel Henry Steel Olcott, William Quan
Judge, and others.
(2) This Movement tended to give the Indians a sense of false pride in their past greatness.
(3) This movement got remarkable success within India as well as outside India.
Which of the statements given above are correct?
a. 1 and 2 only
b. 2 and 3 only
c. 1 and 3 only
d. 1, 2 and 3

Answer: a
Explanation: Olcott was its first president, and remained president until his demise in 1907.
After a few years, Olcott and Blavatsky moved to India and established the International
Headquarters at Adyar, in Madras (now Chennai). Secret Doctrine, is a book written by
Helena Blavatsky. So statement 1 is correct.
The Theosophical Society movement in India tended to give the Indians a sense of false
pride in their past greatness though this movement was not very success within India or
outside of India. So statement 3 is incorrect.
So the right option is a.

Q42. Consider the following statements:


(1) At the Lucknow session, the Indian National Congress and Muslim League sank their
old differences and put up common political demands before the British Government.
(2) The British Government suppressed the publication of the ‘Al-Hilal’ of Abul Kalam Azad
and ‘The Comrade’ of Maulana Mohamed Ali in 1916.
(3) The Lucknow Pact signed by Indian National Congress - Muslim League was facilitated
by Lokmanya Tilak and Mohammed Ali Jinnah.
(4) At the Lucknow session of the Indian National Congress, the moderate and extremist
wings of the Congress were united.
Which of the statements given above are correct?
a. 1 and 2 only
b. 2, 3 and 4 only
c. 1, 2 and 3 only
d. 1, 3 and 4 only

Answer: d
Explanation: The Al-Hilal newspaper by Maulana Abdul Kalam Azad and The Comrade
newspaper by Maulana Mohammad Ali were shut down by the British Government under
the Press Act of 1914. So statement 2 is incorrect. Tilak was released from jail in 1914
and set out to unify two-stream of members of Congress. Jinnah and Tilak believed at that
time that India could win self- government only through Hindu-Muslim unity. So statement
3 is correct. So the right option is d.

Q43. Which of the following statements is incorrect about the Indian Councils Act of 1909?
a. It increased the number of elected members in the Imperial Legislative Council and
Provincial Legislative Councils.
b. They were designed to democratise Indian administration.
c. The reforms in no way changed the undemocratic and foreign character of British rule.
d. The Act introduced the system of separate electorates for Muslims for the first time.

Answer: b
Explanation: It was more commonly called the Morley-Minto Reforms after the Secretary
of State for India John Morley and the Viceroy of India, the 4th Earl of Minto. What the
people of India demanded was that there should be set up a responsible government in the
country. But the sacred heart of the reforms of 1909 was “benevolent despotism” and it was
basically a subtle attempt to create a “constitutional autocracy”. Reforms were never
designed to democratize Indian administration. So statement b is incorrect.

Q44. Consider the following statements regarding the Ghadar Movement:


(1) The Ghadar Movement failed to generate a productive and sustained leadership which
was capable of integrating the various aspects of the Movement.
(2) The Ghadarites consciously promoted communal sentiments among the Punjabis.
Which of the statements given above is/are correct?
a. 1 only
b. 2 only
c. Both 1 and 2
d. Neither 1 nor 2

Answer: a
Explanation: Most of the plans of the Gadarites either failed or were foiled by the British
agents and by the end of February 1915, most of the Ghadar activists were taken captives.
The Special Tribunal prosecuted the Ghadarites. As many as 46 including Kartar Singh
Sarabha and Vishnu Ganesh Pingle were given death sentences, 69 were imprisoned for
life and 125 were given varying terms of imprisonment. Thus it can be said that The Ghadar
Movement failed to generate an effective and sustained leadership which was capable of
integrating the various aspects of the movements. So statement 1 is correct.
The Ghadrites consciously set out to create a secular consciousness among the Punjabis.
No discussion or debate was to take place on religion within the organisation. Religion was
considered a personal matter and that it had no place in the organisation. So statement 2
is incorrect.

Q45. With respect to the Montford Reforms 1919, consider the following statements:
(1) Dyarchy at Provinces was introduced.
(2) Separate electorates for Muslims was continued.
(3) Two Houses of Legislature at Centre was proposed
(4) The Governor could not overrule the Ministers.
Which of the statements given above is/are correct?
a. 1 only
b. 1 and 2 only
c. 1, 3 and 4 only
d. 1, 2 and 3 only

Answer: d
Explanation: Separate electorates for Muslims which were started under Morley-Minto
Reforms, 1909 were continued. So statement 2 is correct. The governor was the executive
head of the provincial government. The subjects were classified into two lists – reserved
and transferred. The reserved list was under the governor and the councillors and the
transferred list was under the ministers. Ministers could not overrule Governor in provinces,
So statement 4 is correct, therefore the correct option is d

Q46. Consider the following statements about the Princely States in India during the 20th
century:
(1) People of the Princely States started organising movements for democratic rights and
popular governments.
(2) The Chamber of Princes first met in February 1921 and all States were assigned full
legislative powers in their States.
Which of the statements given above is/are correct?
a. 1 only
b. 2 only
c. Both 1 and 2
d. Neither 1 nor 2

Answer: a
Explanation: The Chamber of Princes was an institution established by a Royal
Proclamation of the King-Emperor to provide a forum in which the rulers of the Indian
princely states could voice their needs and aspirations to the government of British India.
According to the recommendations of Montford Reforms (1921), a Chamber of Princes
(Narendra Mandal) was set up as a consultative and advisory body having no say in the
internal affairs of individual states and having no powers to discuss matters concerning
existing rights and freedoms. Statement 2 is incorrect.

Q47. The Quit India Movement, devoid of central leadership, branched into a multiplicity of
mini mutinies. Parallel governments were formed in various parts of the country, including
those under the leadership of Chittu Pandey, YB Chavan and others. The place which did
not rise up under a parallel government was:
a. Ballia
b. Satara
c. Mirzapur
d. Tamluk

Answer: c
Explanation: The movement gave the slogans ‘Quit India’ or ‘Bharat Chodo’. Gandhi gave
the slogan to the people – ‘Do or die’. The failure of the Cripps Mission to guarantee any
kind of a constitutional remedy to India’s problems also led to the INC calling for a mass
civil disobedience movement. The chief areas of the movement were UP, Bihar,
Maharashtra, Midnapore and Karnataka. The movement lasted till 1944. During the Quit
India Movement, Parallel governments were formed in various parts of the country in Balia,
Satara and Tamluk, but not in Mirzapur, so correct answer is option c

Q48. Consider the following statements with reference to the nationalists’ response to the
British participation in the World War-I:
(1) The Moderates supported the empire in the war as a matter of duty.
(2) The Extremists including Tilak supported the war efforts in the mistaken belief that Britain
would repay India's loyalty with gratitude in the form of self-government.
(3) The revolutionaries decided to utilize their power to support the War.
(4) The Indian supporters of British war efforts failed to see that the imperialist powers were
fighting precisely to safeguard their own colonies and markets.
Which of the statements given above are correct?
a. 1, 2 and 3 only
b. 2, 3 and 4 only
c. 1, 2 and 4 only
d. 1, 2, 3 and 4

Answer: c
Explanation: The outbreak of the First World War in 1914 gave a new lease of life to the
nationalist movement since Britain's difficulty was seen as India's opportunity by the
revolutionaries. This opportunity was seized, in different ways and with varying success, by
the Ghadar revolutionaries based in North America. After the outbreak of World War-I,
Ghadar Party members returned to Punjab and in 1915, they conducted revolutionary
activities in central Punjab and organised uprisings. So statement 3 is incorrect.

Q49. Which one of the statements given below is not true about the Swaraj Party
constituted by Chittaranjan Das and Motilal Nehru?
a. Chittaranjan Das and Motilal Nehru resigned from their offices in the All India National
Congress to constitute Congress-Khilafat Swaraj Party.
b. The Swaraj Party declared to present the national demand for self-government in the
Legislative Councils under the British rule.
c. The Swarajists stated that participation in the Legislative Councils under the British rule
was necessary to occupy the place of power, which was provided to the Councils by the
Mont-Ford reforms.
d. The Swarajists lacked any policy of coordinating their militant work in the Legislative
Councils with mass political work outside.

Answer: c
Explanation: The Swaraj Party won 42 out of 104 seats to the Central Legislature in 1923.
Swarajist Vithalbhai Patel became speaker of the Central Legislative Assembly in 1925.
They outvoted the government many times even in matters related to budgetary grants. The
Swarajists stated that to participate in the legislative councils under the British rule, was
necessary to fill the concurrent temporary political gap after the Non-cooperation Movement
and to keep up the morale of the politicized Indian. So statement c is not correct.
Chittaranjan Das and Motilal Nehru resigned from their offices in the All India National
Congress to constitute Congress-Khilafat Swaraj Party so statement a is correct.

Q50. With reference to the Simon Commission consider the following statements:
(1) The Simon Commission was appointed in 1927 and it submitted its report in 1929.
(2) The Government of India Act, 1935 was passed by the British Parliament according to
the recommendations of the Simon Commission without any amendments.
Which of the statements given above is/are correct?
a. 1 only
b. 2 only
c. Both 1 and 2
d. Neither 1 nor 2
Answer: d
Explanation: The commission submitted its report in 1930. So statement 1 is incorrect.
Noting that the educated Indians opposed the Simon Commission Report, the British
government decided that before the publication of the report, the Indian opinion would be
taken into account. So statement 2 is also not true.
The correct answer is option d.

Q51. Consider the following statements regarding Salt Satyagraha:


(1) The salt satyagraha began on March 12, 1930, and ended in Dandi with Gandhi breaking
the Salt Act on April 6, 1930.
(2) Gandhi chose April 6 to launch the mass breaking of the salt laws for a symbolic
reason— it was the first day of “National Week”, begun in 1919 when Gandhi conceived of
the national hartal (strike) against the Rowlatt Act.
(3) Instead of choosing the direct route from Ahmedabad to Dandi, Gandhiji chose to pass
through areas under the rulers of the princely states to avoid arrest.
(4) Lord Reading was the British Viceroy during Salt Satyagraha.
Which of the statements given above are correct?
a. 1 and 2 only
b. 2, 3 and 4 only
c. 3 and 4 only
d. 1, 2, 3 and 4

Answer: a
Explanation: The 24-day march lasted from 12 March 1930 to 6 April 1930 as a direct
action campaign of tax resistance and nonviolent protest against the British salt monopoly.
Instead of choosing the direct route from Ahmedabad to Dandi, Gandhiji chose to pass
through areas which were under British India to avoid discomfort to the princely rulers of
that area. So statement 3 is incorrect.
Lord Irwin was the British Viceroy during Salt Satyagraha or Civil Disobedience Movement,
so statement 4 is incorrect. So the right answer is a as both statements 1 and 2 are
correct.

Q52. Which among the following statements about a Constitution is incorrect?


a. A Constitution is a body of fundamental principles according to which a State is
constituted or governed.
b. The Japanese Constitution of 1947 is popularly known as the ‘peace constitution.’
c. An essential function of a Constitution is to specify who has the power to make decisions
in a society.
d. In democratic constitutions, one single party is given the power to decide.

Answer: d
Explanation:
The first function of a Constitution is to provide a set of basic rules that allow for minimal
coordination amongst members of society. It specifies the necessary allocation of power in
a society. It decides who gets to determine what the laws will be. In principle, this question,
who gets to decide, can be answered in many ways: in a monarchical constitution, a
monarch decides; Statements a, b and c are correct, whereas statement d is incorrect
as in some Constitutions like the old Soviet Union, one single party was given the power to
decide. However, generally, in democratic constitutions, the people get to decide.

Q53. Consider the following pairs:

Features Sources
Borrowed
1. Independent : US
Judiciary Constitution
2. Law Making : Australian
Procedure Constitution
3. Institution of : British
the Speaker Constitution

Which of the pairs given above is/are correctly matched?


a. 1 and 3 only
b. 1 only
c. 2 and 3 only
d. 1, 2 and 3

Answer: a
Explanation: Options 1 and 3 are correct.
US Constitution - Power of Judicial Review and independence of the judiciary.
British Constitution - Institution of the Speaker and his role in Lawmaking procedure
Option 2 is incorrect.
Provisions related to Lawmaking procedure is not borrowed from the Australian
Constitution.

Q54. Consider the following statements:


(1) The Indian Constitution makes ethnic identity a criterion for citizenship.
(2) The United Kingdom has a written constitution.
Which of the statements given above is/are incorrect?
a. 1 only
b. 2 only
c. Both 1 and 2
d. Neither 1 nor 2

Answer: c
Explanation: The Indian Constitution does not make ethnic identity a criterion for
citizenship. Most importantly, a Constitution expresses the fundamental identity of a people.
This means the people as a collective entity comes into being only through the fundamental
Constitution. It is by agreeing to a basic set of norms about how one should be governed,
and who should be governed that one forms a collective identity.
In most countries, ‘Constitution’ is a compact document that comprises several articles,
specifying how the state is to be constituted and what norms it should follow. However,
some countries, like the United Kingdom, do not have one single document that can be
called the Constitution. Instead, they have a series of documents and decisions that, taken
collectively, are referred to as the Constitution, so the Constitution of UK is considered as
an unwritten Constitution.

Q55. Regarding the Constituent Assembly of India, consider the following statements:
(1) It was constituted under the scheme formulated by the Cabinet Mission Plan.
(2) The Constituent Assembly passed all the provisions of the Constitution only after
debating every provision.
(3) All members were elected directly by the members of the Provincial Legislative
Assemblies.
Which of the statements given above is/are incorrect?
a. 1 and 2 only
b. 2 and 3 only
c. 3 only
d. 1, 2 and 3

Answer: b
Explanation:
Statement 1 is correct.
The Constituent Assembly was composed roughly along the lines suggested by the Plan
proposed by the Committee of the British Cabinet, known as the Cabinet Mission.
According to this plan:
Each Province and each Princely State or group of States were allotted seats proportional
to their respective population roughly in the ratio of 1:10,00,000. As a result, the Provinces
(that were under direct British rule) were to elect 292 members while the Princely States
were allotted a minimum of 93 seats.
The seats in each Province were distributed among the three central communities, Muslims,
Sikhs and general, in proportion to their respective populations.
Members of each community in the Provisional Legislative Assembly elected their
representatives by the method of proportional representation with a single transferable vote.
The method of selection in the case of representatives of Princely States was to be
determined by consultation.
Statement 2 is incorrect.
In the Constituent Assembly only one provision of the Constitution was passed without
virtually any debate: the introduction of universal suffrage (meaning that all citizens
reaching a certain age would be entitled to become voters irrespective of religion, caste,
education, gender or income). So, while the members felt no need at all to discuss the issue
of who should have the right to vote, every other matter was seriously discussed and
debated.
Statement 3 is incorrect.
The members of the Constituent Assembly were elected by indirect election by the
members of the Provincial Legislative Assemblies. The members to the Provincial
Legislative Assemblies had been elected in early January 1946.

Q56. Consider the following provisions borrowed from the Constitutions of different
countries:
(1) First Past the Post
(2) Parliamentary Form of Government
(3) The idea of the rule of law
(4) A quasi-federal form of government (a federal system with a strong central government)
(5) The idea of Residual Powers
Which of the above provisions are not borrowed from the Canadian Constitution?
a. 1, 2 and 3 only
b. 1, 3, 4 and 5 only
c. 1, 2, 3 and 4 only
d. 4 and 5 only

Answer: a
Explanation: Indian Constitution has borrowed following features from the Canadian
Constitution - A quasi-federal form of government (a federal system with a strong central
government) and the idea of Residual Powers. First Past the Post, Parliamentary Form of
Government, The idea of the rule of law, Institution of the Speaker and his role and
Lawmaking procedure has been borrowed from the British Constitution. So only 4 and 5 are
taken from the Canadian Constitution. So the correct option is c.

Q57. Which among the following statements about the composition of the Constituent
Assembly of India is correct?
a. Two hundred ninety-nine members of the Constituent Assembly were present on 26
November 1949 and appended their signature to the Constitution as finally passed.
b. In the Indian Constitution, religious identity has an essential bearing on citizenship rights.
c. The Assembly had twenty-six members from what was then known as the Scheduled
Classes.
d. In terms of political parties, the Congress dominated the Assembly occupying as many
as fifty per cent of the seats in the Assembly after the Partition.

Answer: c
Explanation: As a consequence of the Partition under the plan of 03 June 1947, those
members who were elected from territories which fell under Pakistan ceased to be members
of the Constituent Assembly. The numbers in the Assembly were reduced to 299 of which
284 were present on 26 November 1949 and appended their signature to the Constitution
as finally passed. So statement a is not correct.
Even though the Indian Constitution was framed against the backdrop of the horrendous
violence that the Partition unleashed on the sub-continent, the Constitution was committed
to a new conception of citizenship, where not only would minorities be secure, but religious
identity would have no bearing on citizenship rights. So statement b is incorrect.
In terms of political parties, the Congress dominated the Assembly occupying as many as
eighty-two per cent of the seats in the Assembly after the Partition. So statement d is
incorrect.
The Assembly had twenty-six members from what was then known as the Scheduled
Classes. So the only correct statement is c.

Q58. Consider the following statements:


(1) Special Majority and people’s participation are two principles that dominate the various
procedures of amending the Constitutions in most modern countries.
(2) The Constitutions of the U.S., South Africa, Russia, have employed people’s
participation in the process of amending the Constitution principle.
(3) In Switzerland, people can even initiate an amendment to the Constitution.
Which of the statements given above is/are incorrect?
a. 1 only
b. 1 and 2 only
c. 2 only
d. 2 and 3 only

Answer: c
Explanation: Two principles dominate the various procedures of amending the
Constitutions in most modern countries. One is the principle of a special majority. For
instance, the Constitutions of the U.S., South Africa, Russia, India etc. have employed this
principle. In the case of the Constitution of India and the US, it is a two-thirds majority, while
in South Africa and Russia, for some amendments, a three-fourths majority is required. The
other principle that is popular is that of people’s participation in the process of amending
the Constitution, especially in modern countries. In Switzerland, people can even initiate an
amendment to the Constitution. Other examples of countries where people initiate or
approve an amendment to the Constitution are Russia and Italy, among others.

Q59. Which of the following was part of the historic Objectives Resolution?
(1) All people of India shall be guaranteed and secured social, economic and political
justice.
(2) India is an Independent, Secular and Republic country.
(3) All powers and authority of independent India and its Constitution shall flow from the
people.
Select the correct answer using the code given below:
a. 1 and 2 only
b. 2 and 3 only
c. 1 and 3 only
d. 1, 2 and 3

Answer: c
Explanation: The Objectives Resolution encapsulated the aspirations and values behind
the Constitution. Based on this resolution, our Constitution gave institutional expression to
these fundamental commitments: equality, liberty, democracy, sovereignty and
cosmopolitan identity.
Statements 1 and 3 are correct.
Main points of the Objectives Resolution
India is an independent, sovereign republic. Hence statement 2 is incorrect as the
word secular was not used at that time. (It was introduced by 42nd constitutional
amendment)
India shall be a Union of erstwhile British Indian territories, Indian States, and other parts
outside British India and the such Indian States as are willing to be a part of the Union
Territories forming the Union shall be autonomous units and exercise all powers and
functions of the Government and administration, except those assigned to or vested in the
Union
All powers and authority of sovereign and independent India and its Constitution
shall flow from the people
All people of India shall be guaranteed and secured social, economic and political
justice; equality of status and opportunities and equality before the law; and fundamental
freedoms - of speech, expression, belief, faith, worship, vocation, association and action -
subject to the law and public morality.
The minorities, backward and tribal areas, depressed and other backward classes shall be
provided adequate safeguards.
The territorial integrity of the Republic and its sovereign rights on land, sea and air shall be
maintained according to justice and law of civilised nations.
The land would make a full and willing contribution to the promotion of world peace and
welfare of humanity.

Q60. The provisions of equality of status and equality of opportunity have been provided in
a. Fundamental Rights
b. Directive Principles of State Policy
c. The Preamble
d. Directive Principles of State Policy and the Preamble

Answer: c
Explanation: The Preamble to the Indian Constitution is based on the ‘Objectives
Resolution’, drafted and moved by Jawaharlal Nehru, and adopted by the Constituent
Assembly. It has been amended by the 42nd Constitutional Amendment Act (1976), which
added three new words—socialist, secular and integrity
Preamble mentions two things about equality:
Equality of status
Equality of opportunity- here equality of opportunity means that all sections of the society
enjoy equal opportunities

Q61. Consider the following statements concerning liberalism in India:


(1) The liberalism of the Indian Constitution was always linked to social justice; the best
example of this is the provision for reservations for Scheduled Castes and Scheduled Tribes
in the Constitution.
(2) Rammohan Roy emphasised individual rights, particularly the rights of women, which is
one of the streams of Indian liberalism.
Which of the statements given above is/are correct?
a. 1 only
b. 2 only
c. Both 1 and 2
d. Neither 1 nor 2

Answer: c
Explanation: Liberalism broadly encompasses three dimensions- a economic liberalism,
which emphasises free competition and the self-regulating market, i.e., minimal state
intervention in the economy. b political liberalism, which is founded on the belief in progress,
the autonomy of the individual, and their political and civil liberties c social liberalism, which
is linked to the protection of minority groups, and such issues as LGBTQ rights and same-
sex marriage.
The liberalism of the Indian Constitution differs from classical western liberalism in the
sense that western-style liberalism prioritises individual rights over those of society; while
for Indians, demands of social justice and community values are more important. The
makers of the Constitution believed that the mere granting of individual rights such as the
right to equality were not enough to overcome age-old injustices suffered by these groups
or to give real meaning to their right to vote. Special constitutional measures were required
to advance their interests. Therefore, the Constitution-makers provided several special
measures to protect the interests of SC/STs, such as the reservation of seats in legislatures.
Indian liberalism has two streams. The first stream began with Rammohan Roy. He
emphasised on individual rights, particularly the rights of women. The second stream
included thinkers like K.C. Sen, Justice Ranade and Swami Vivekananda. They introduced
the spirit of social justice within orthodox Hinduism. So statement 2 is correct.

Q62. Consider the following statements:


(1) Freedom of Religion includes the freedom not to follow any religion.
(2) The government cannot impose any restrictions on the practice of Freedom of Religion.
(3) The government can discriminate based on religion in providing public employment.
Which of the statements given above is/are incorrect?
a. 1 and 2 only
b. 2 and 3 only
c. 1 only
d. 1, 2 and 3

Answer: b
Explanation:
According to our Constitution, everyone enjoys the right to follow the religion of his or her
choice. This freedom is considered as a hallmark of democracy.
Statement 1 is correct.
Freedom of religion also includes freedom of conscience. This means that a person may
choose any religion or may choose not to follow any religion. Freedom of Religion includes
the freedom to profess, follow and propagate any religion
Statement 2 is incorrect.
Freedom of Religion is subject to certain limitations. The government can impose
restrictions on the practice of freedom of religion in order to protect public order, morality
and health. This means that the freedom of religion is not an unlimited right. The government
can interfere in religious matters for rooting out certain social evils.
Statement 3 is incorrect.
India does not have any official religion. We do not have to belong to any particular religion
in order to be the prime minister or president or judge or any other public official. Also, under
the right to equality, there is a guarantee that the government will not discriminate based on
religion in giving employment.

Q63. In so far as the Constitution of India is concerned, minority status is recognised on


which of the following grounds?
(1) Religion
(2) Language
(3) Culture
Select the correct answer using the code given below:
a. 1 and 2 only
b. 2 and 3 only
c. 1 and 3 only
d. 1, 2 and 3

Answer: d
Explanation:
All options are correct.
Our Constitution believes that diversity is our strength. Therefore, one of the fundamental
rights is the right of minorities to maintain their culture. This minority status is not
dependent only upon religion. Linguistic and cultural minorities are also included in
this provision. Minorities are groups that have common language or religion, and in a
particular part of the country or the country as a whole, they are outnumbered by some
other social section. Such communities have a culture, language and a script of their own,
and have the right to conserve and develop these.

Q64. Which of the following provisions under the Constitution of India has been recognised
as “Heart and Soul of the Constitution”?
a. Right to Life and Right to Constitutional Remedy
b. Right to Constitutional Remedy
c. Directive Principles of State Policy
d. Preamble and the Fundamental Duties

Answer: b
Explanation: Dr. Ambedkar considered the right to constitutional remedies as ‘heart
and soul of the constitution’. It is so because this right gives a citizen the right to approach
a High Court or the Supreme Court to get any of the fundamental rights restored in case of
their violation. The Supreme Court and the High Courts can issue orders and give directives
to the government for the enforcement of rights.

Q65. Consider the following statements about Fundamental Rights:


(1) It prohibits the government from acting against the rights of the individuals, but in case
there is a violation of these rights, nothing is explicitly mentioned in the Indian Constitution.
(2) Article 14 of the Constitution explicitly clarifies that a policy like reservation will not be
seen as a violation of the right to equality.
Which of the statements given above is/are correct?
a. 1 only
b. 2 only
c. Both 1 and 2
d. Neither 1 nor 2
Answer: d
Explanation: Fundamental Rights prohibit the government from acting against the rights of
the individuals and in case there is a violation of these rights, Article 32 mentioned in the
Indian Constitution gives rights to individuals to approach the Supreme and the High Courts.
The Courts can issue writs to hold upright the basic fundamental rights. So statement 1 is
incorrect.
The Constitution clarifies that the government can implement special schemes and
measures for improving the conditions of individual sections of society: children, women,
and the socially and educationally backward classes. Article 16(4) of the Constitution
explicitly clarifies that a policy like reservation will not be seen as a violation of the right to
equality. So statement 2 is incorrect.

Q66. Concerning Article 21, which among the following statements is incorrect?
a. No citizen can be denied his or her life except by procedure as laid down under the law.
b. Protection against arrest and detention is also part of this right.
c. The Supreme Court has ruled that this right also includes the right to live with human
dignity, free from exploitation.
d. Right to shelter and livelihood is included in the right to life.

Answer: b
Explanation: Article 21 advocates protection of life and personal liberty i.e. No person shall
be deprived of his life or personal liberty except according to the procedure established by
law. So statement a is correct.
Article 22 envisages protection against arrest and detention in some instances. So
statement b is not correct.
The SC has held that the right to life as enshrined in Article 21 means something more than
survival or animal existence and would include the right to live with human dignity. Thus,
the Supreme Court has time and again expanded the scope of Article 21. So Statement c
is also correct.
The Court, in Olga Tellis vs BMC case 1985, has held that right to shelter and livelihood is
also included in the right to life because no person can live without the means of living, that
is, the means of livelihood. So statement d is correct.

Q67. Which among the following statements with regards to the right to property in the
Indian Constitution is correct?
a. In the Constitution, initially, there was no fundamental right to property; it was inserted
later.
b. The Constitution made it clear that property could be taken away by the government for
public welfare.
c. In 1973, the Supreme Court gave a decision that the right to property was part of the
basic structure of the Constitution.
d. In 1976, the 42nd Constitutional Amendment to the Constitution removed the right to
property from the list of Fundamental Rights.

Answer: b
Explanation: In the Constitution, initially, there was a fundamental right to ‘acquire, possess
and maintain’ property under Article 19 (1)(f). So statement a is incorrect.
However, the Constitution made it clear that property could be taken away by the
government for public welfare. Since 1950, the government has made many laws that have
limited this right to property. Therefore statement b is correct.
In 1973, the Supreme Court gave a decision that the right to property was not part of the
basic structure of the Constitution, and therefore, the Parliament had the power to abridge
this right by an amendment. So statement c is incorrect.
In 1978, the 44th amendment to the Constitution removed the right to property from the list
of Fundamental Rights and converted it into a simple legal right under Article 300A.
Statement d is incorrect.

Q68. Consider the following statements about writs:


(1) The Writ of Prohibition is issued by a higher court to a lower court or another authority
to transfer a matter pending before it to the higher authority or court.
(2) If the court finds that a person is holding office but is not entitled to hold that office, the
court issues the writ of quo warranto and restricts that person from acting as an officeholder.
(3) Under the writ of Certiorari, the higher court asks a lower court to transfer the case as it
has considered a case going beyond its jurisdiction.
(4) Sardar Vallabh Bhai Patel considered the right to constitutional remedies as ‘heart and
soul of the Constitution’.
Which of the statements given above is/are incorrect?
a. 1, 2 and 3 only
b. 2, 3 and 4 only
c. 1, 3 and 4 only
d. 1, 2 and 4 only

Answer: c
Explanation: Article 32 gives a citizen the right to approach a High Court or the Supreme
Court to get any of the fundamental rights restored in case of their violation. The courts can
issue various special orders known as writs - Habeas corpus, Mandamus, Prohibition,
Quo Warranto, Certiorari. Prohibition writ is issued by a higher court (High Court or
Supreme Court) when a lower court has considered a case going beyond its jurisdiction. So
statement 1 is incorrect.
If the court finds that a person is holding office but is not entitled to hold that office, it issues
the writ of quo warranto and restricts that person from acting as an officeholder. So
statement 2 is correct.
The writ of certiorari can be issued by the Supreme Court or any High Court for quashing
the order already passed by an inferior court, tribunal or quasi-judicial authority. Therefore
statement 3 is incorrect. Statement 4 is attributed to Dr. B.R. Ambedkar, so statement 4
is also incorrect.
Q69. About Fundamental Duties in the Indian Constitution, which of the statements given
below are correct?
(1) In 1976, Fundamental Duties in the Indian Constitution were inserted.
(2) The 42nd Constitutional Amendment to the Constitution introduced a list of Fundamental
Duties for citizens.
(3) There are eleven Fundamental Duties that the Constitution has enumerated and
discussed the procedure to enforce these duties.
(4) The inclusion of Fundamental Duties has brought a sea change in the status of our
Fundamental Rights.
Select the correct answer using the code given below:
a. 1 and 2 only
b. 2 and 3 only
c. 3 and 4 only
d. 1 and 4 only

Answer: a
Explanation: In 1976, the 42nd amendment to the Constitution was passed. Among other
things, this amendment inserted a list of Fundamental Duties for citizens. Thus, statements
1 and 2 are correct.
In all, eleven duties were enumerated. However, the Constitution does not say anything
about enforcing these duties. So, statement 3 is incorrect.
However, it must be noted that our Constitution does not make the enjoyment of rights
dependent or conditional upon fulfilment of duties. In this sense, the inclusion of
Fundamental Duties has not changed the status of our Fundamental Rights. So statement
4 is incorrect.

Q70. Which among the following statements is/are correct?


(1) Guidelines which were incorporated in the Indian Constitution but made non-justiciable
are called the Directive Principles of State Policy.
(2) Governmental efforts to give effect to the Directive Principles include the Right to
Education, the formation of Panchayati Raj institutions all over the country, partial right to
work under the Employment Guarantee Programme and the Mid-day Meal Scheme etc.
Select the correct answer using the code given below:
a. 1 only
b. 2 only
c. Both 1 and 2
d. Neither 1 nor 2

Answer: c
Explanation: If a government did not implement a particular Directive Principle of State
Policy, we could not go to the court asking the court to instruct the government to implement
that policy. Thus, these guidelines are ‘non-justiciable,’ i.e., parts of the Constitution that
cannot be enforced by the judiciary. So statement 1 is correct.
The governments from time to time tried to give effect to some Directive Principles of State
Policy. They passed several zamindari abolition bills, nationalised banks, enacted
numerous factory laws, fixed minimum wages, cottage and small industries were promoted
and provisions for reservation for the upliftment of the scheduled castes and scheduled
tribes were made. Such efforts to give effect to the Directive Principles include the right to
education, the formation of Panchayati raj institutions all over the country, partial right to
work under employment guarantee programme and the mid-day meal scheme etc. So
statement 2 is correct.

Q71. Consider the following statements concerning the basic structure of the Indian
Constitution:
(1) The theory of the basic structure of the Constitution was advanced by the lower Judiciary
and the Parliament.
(2) It has set specific limits to the State Legislative Assemblies' power to amend the
Constitution, leaving aside the Parliament free to amend the Constitution.
Which of the statements given above is/are correct?
a. 1 only
b. 2 only
c. Both 1 and 2
d. Neither 1 nor 2

Answer: d
Explanation: One thing that has had a long-lasting effect on the evolution of the Indian
Constitution is the Theory Of The Basic Structure of the Constitution. The Supreme Court
of India advanced this theory in the famous case of Kesavananda Bharati vs State of
Kerala, 1973. So statement 1 is incorrect.
This ruling has contributed to the evolution of the Constitution in the following ways: It has
set specific limits to the Parliament’s power to amend the Constitution. It says that no
amendment can violate the basic structure of the Constitution. It also places the Judiciary
as the final authority in deciding if an amendment violates basic structure and what
constitutes the basic structure. So statement 2 is also incorrect as the State legislative
Assemblies are not mentioned.
The Supreme Court is yet to define what constitutes the basic structure of the Constitution.
However, in general, these are those core structure of Indian constitution on which the
whole superstructure of the Constitution is built. Without being exhaustive, these basic
features are- Parliamentary System, Democratic Republic, Supremacy of Constitution, Rule
of Law, Separation of Power, Federalism, Free and Fair Election, Dignity of Individual,
Socio-economic Justice etc.

Q72. Consider the following statements:


(1) The authority to enact laws and policies is bestowed on the Parliament at central level
by the Indian Constitution.
(2) The Constitution limits the power of the government by providing certain Fundamental
(3) Rights for citizens which the government cannot violate except under certain
circumstances.
Which of the statements given above is/are correct?
a. 1 only
b. 2 only
c. Both 1 and 2
d. Neither 1 nor 2

Answer: c
Explanation: If the Parliament has the authority to enact laws, there must be a law that
bestows this authority on the Parliament in the first place. This is the function of the
Constitution. It is an authority that constitutes the government in the first place. So
Statement 1 is correct.
A vital function of a Constitution is to set some limits on what a government can impose on
its citizens. These limits are fundamental in the sense that the government may never
trespass them, i.e. fundamental rights. The exact content and interpretation of these rights
vary from the constitution to constitution. However, most constitutions will protect a primary
cluster of rights. So statement 2 is correct.

Q73. The Parliament ensures accountability of the Executive through


(1) Deliberation and Discussion via the Question Hour
(2) By Ratification of Laws
(3) Financial control by approval and passing of the Budget.
Which of the statements given above are correct?
a. 1 and 2 only
b. 2 and 3 only
c. 1 and 3 only
d. 1, 2 and 3

Answer: d
Explanation:
All statements are correct.
The Legislature in the parliamentary system ensures Executive accountability at various
stages: policymaking, implementation of law or policy, and during and post-implementation
stage. The Legislature does this through the use of a variety of devices:
Deliberation and discussion. During the law-making process, members of the legislature
get an opportunity to deliberate on the policy direction of the executive and how policies are
implemented. The Question Hour, which is held every day during the sessions of
Parliament, where Ministers have to respond to searching questions raised by the
members; Zero Hour where members are free to raise any matter that they think is
important (though the ministers are not bound to reply), half-an-hour discussion on
matters of public importance, adjournment motion etc. are some instruments of
exercising control
Approval or Refusal of laws. Parliamentary control is also exercised through its power of
ratification. A bill can become a law only with the approval of the Parliament.
Financial control. Since the financial resources to implement the programmes of the
government are granted through the budget. Preparation and presentation of the budget
for the approval of the legislature is the constitutional obligation of the government.
No-Confidence Motion. The most potent weapon that enables the Parliament to ensure
executive accountability is the no-confidence motion. As long as the government has the
support of its party or coalition of parties that have a majority in the Lok Sabha, the power
of the House to dismiss the government is fictional rather than real.

Q74. Consider the following statements:


(1) Both the Lok Sabha and the Rajya Sabha can initiate the procedure for removal of the
Vice President.
(2) The Rajya Sabha can suggest amendments to the Money Bill.
(3) The Union Council of Ministers is responsible to the Parliament.
Which of the statements given above is/are incorrect?
a. 1 and 2 only
b. 2 and 3 only
c. 1 and 3 only
d. 1, 2 and 3

Answer: c
Explanation:
Statement 1 is incorrect.
Powers of Rajya Sabha
It approves constitutional amendments.
Exercises control over executive by asking questions, introducing motions and resolutions.
Participates in the election and removal of the President, Vice President, Judges of
Supreme Court and High Court.
It can alone initiate the procedure for removal of Vice President.
It can also give, Parliament the power to make laws on matters included in the State list
It considers and approves non money bills and suggests amendments to money bills.
Hence statement 2 is correct.
Statement 3 is incorrect.
The Rajya Sabha cannot initiate, reject or amend money bills. The Council of Ministers is
responsible to the Lok Sabha and not Rajya Sabha. Therefore, Rajya Sabha can criticise
the government but cannot remove it.
Q75. The members of the Rajya Sabha can be nominated from which of the following fields?
(1) Social Service
(2) Co-operative Movement
(3) Literature
(4) Science
Select the correct answer using the code given below:
a. 1, 2 and 3 only
b. 2 and 4 only
c. 1, 3 and 4 only
d. 1, 2, 3 and 4

Answer: c
Explanation:
Option 1, 3 and 4 are correct.
Apart from the elected members, Rajya Sabha also has twelve nominated members. The
President nominates these members. These nominations are made from among those
persons who have made their mark in the fields of literature, arts, social service, science
etc
Option 2 is incorrect.
The members of the Legislative Council to be nominated by the Governor shall consist of
persons having special knowledge or practical experience in literature, arts, social service
and the cooperative movement.

Q76. Concerning the President of India, consider the following statements:


(1) S/he cannot withhold or refuse to give assent to Bills other than Money Bills.
(2) S/he can only be impeached on the ground of violation of the Constitution.
Which of the statements given above is/are correct?
a. 1 only
b. 2 only
c. Both 1 and 2
d. Neither 1 nor 2

Answer: b
Explanation:
Statement 1 is incorrect.
The President also has veto power by which s/he can withhold or refuse to give assent to
Bills (other than Money Bill) passed by the Parliament. Every bill passed by the Parliament
goes to the President for his assent before it becomes a law. The President can send the
bill back to the Parliament asking it to reconsider the bill. This ‘veto’ power is limited
because, if the Parliament passes the same bill again and sends it back to the President,
then, the President has to give assent to that bill.
Statement 2 is correct.
The President can be removed from office only by Parliament by following the procedure
for impeachment. This procedure requires a special majority. The only ground for
impeachment is a violation of the Constitution.

Q77. Regarding the Constitution of India, consider the following statements:


(1) The executive power of the Union vests in the Prime Minister.
(2) According to the 91st Constitutional Amendment, the size of the Union Council of
Ministers shall not exceed 15 per cent of the total members of the lower house.
(3) The Vice-President shall be elected by the elected members of both Houses of the
Parliament and the State Legislative Assemblies.
Which of the statements given above is/are incorrect?
a. 1 and 2 only
b. 1 and 3 only
c. 3 only
d. 1, 2 and 3

Answer: b
Explanation:
Statement 1 is incorrect.
The Constitution of India vests the executive power of the Union formally in the President.
In reality, the President exercises these powers through the Council of Ministers headed by
the Prime Minister. The President is elected for five years.
Statement 2 is correct.
The 91st Constitutional Amendment in 2003 made that the Council of Ministers shall not
exceed 15 per cent of the total number of members of the House of People (or Assembly,
in the case of the States).
Statement 3 is incorrect.
The Vice President is elected for five years. His election method is similar to that of the
President; the only difference is that members of State legislatures are not part of the
Electoral College. The Vice President may be removed from his office by a resolution of
the Rajya Sabha passed by a majority of all the then members and agreed to by the Lok
Sabha.

Q78. Consider the following statements:


(1) The Constitution does not provide for reservation of seats in the Lok Sabha for
Scheduled Castes and Scheduled Tribes.
(2) The minimum age to contest the State Assembly election is 25 years.
Which of the statements given above is/are correct?
a. 1 only
b. 2 only
c. Both 1 and 2
d. Neither 1 nor 2

Answer: b
Explanation:
Statement 1 is incorrect.
There are certain social groups which may be spread across the country. In a particular
constituency, their numbers may not be sufficient to be able to influence the victory of a
candidate. However, taken across the country they are a significantly sizeable group. To
ensure their proper representation, a system of reservation becomes necessary. The
Constitution provides for reservation of seats in the Lok Sabha and State Legislative
Assemblies for the Scheduled Castes and Scheduled Tribes.
Statement 2 is correct.
In order to stand/contest for the Lok Sabha or Assembly election, a candidate must be at
least 25 years old.

Q79. With reference to the Election Commission of India, consider the following statements:
(1) To ensure a free and fair election, the Commission can cancel the polls in the entire
country.
(2) It accords recognition and allots symbols to the political parties.
Which of the statements given above is/are correct?
a. 1 only
b. 2 only
c. Both 1 and 2
d. Neither 1 nor 2

Answer: c
Explanation:
Statement 1 is correct.
The Election Commission has the power to take decisions to ensure a free and fair poll.
It can postpone or cancel the election in the entire country or a specific State or
constituency because the atmosphere is vitiated and therefore, a free and fair election may
not be possible. The Commission also implements a model code of conduct for parties and
candidates.
Statement 2 is correct.
The Election Commission accords recognition to political parties and allots symbols
to each of them.
Other than that, the commission is also responsible for preparing of electoral rolls, and issue
electors photo identity cards (EPIC), has advisory jurisdiction in post-election
disqualification of sitting members of Parliament and State Assemblies.

Q80. The Election Commission is not responsible for the conduct of elections of which of
the following?
a. Chairman of the Rajya Sabha
b. Local body elections
c. The President
d. All elections to the Parliament

Answer: b
Explanation: The superintendence, direction and control of the preparation of the electoral
rolls for, and the conduct of, all elections to Parliament and to the Legislature of every
State and of elections to the offices of President and Vice President (Chairman of Rajya
Sabha) held under this Constitution shall be vested in an Election Commission .
Option b is incorrect.
The Election Commission is not responsible for the conduct of local body elections. State
Election Commissions are responsible for local body election.

Q81. Consider the following statements regarding the Higher Judiciary in India:
(1) The Executive does not have any role in the appointment of judges.
(2) The Legislature has the powers to remove the judges.
Which of the statements given above is/are correct?
a. 1 only
b. 2 only
c. Both 1 and 2
d. Neither 1 nor 2
Answer: b
Explanation: Statement 1 is incorrect. The appointments in the higher judiciary are
carried out by the executive based on recommendations by the collegiums consisting of 4
senior-most judges of the Supreme Court. According to the Memorandum of Procedure, the
centre can return the names recommended by the collegiums with due justification. If
collegiums send it back, the executive has to appoint. Thus, the appointment is made by
the Executive in concurrence with the Judiciary.
Statement 2 is correct. The Constitution makes a rigorous procedure for the removal of
the judges of the High Court and the Supreme Court. Each House of Parliament needs to
pass a resolution requiring 2/3rd of the members present and voting as well as more than
the majority of the total membership of the House. The Resolution so passed goes to the
President who orders the removal of the judge. Thus the power of removal is accorded to
the Legislature.

Q82. Consider the following statements about Indian Judiciary:


(1) The Constitution of India provides for a single integrated judicial system with Supreme
Court at the top.
(2) The lower courts do not function under the direct superintendence of the higher courts,
but under the Supreme Court directly.
Which of the statements given above is/are correct?
a. 1 only
b. 2 only
c. Both 1 and 2
d. Neither 1 nor 2

Answer: a
Explanation: The Constitution of India provides for a single integrated judicial system. This
means that unlike some other federal countries of the world, India does not have separate
State courts. The structure of the judiciary in India is pyramidal with the Supreme Court at
the top, High Courts below them and district and subordinate courts at the lowest level. So
statement 1 is correct.
The lower courts function under the direct superintendence of the higher courts, not the
Supreme Court. So, statement 2 is incorrect.

Q83. With regard to the jurisdiction of Supreme Court of India, consider the following
statements:
(1) Under its Original Jurisdiction, the Supreme Court tries appeals from lower courts in
Civil, Criminal and Constitutional cases
(2) Under its Appellate Jurisdiction, the Supreme Court of India settles disputes between
the Union and the States and amongst the States.
(3) Under its Advisory Jurisdiction, the Supreme Court advises the President and other
constitutional offices on matters of public importance and law.
Which of the statements given above is/are incorrect?
a. 1 and 3 only
b. 2 and 3 only
c. 1 and 2 only
d. 1, 2 and 3

Answer: d
Explanation: There are primarily 4 types of Jurisdiction of Supreme Court of India as per
the Indian Constitution - original, appellate, advisory and writ.
Under Original Jurisdiction, Supreme Court of India settles disputes between Union and
States and amongst States. So statement 1 is incorrect.
Under Appellate Jurisdiction, Supreme Court tries appeals from lower courts in Civil,
Criminal and Constitutional cases. So statement 2 is incorrect.
Under Advisory Jurisdiction, Supreme Court advises the President on matters of public
importance and law. So statement 3 is also incorrect. Supreme Court, under article 143,
advises only the President of India.
Under Writ jurisdiction, Supreme Court can issue writs of Habeas Corpus, Mandamus,
Prohibition, Certiorari and Quo warranto to protect the Fundamental Rights of the individual.

Q84. Consider the following statements:


(1) The Lok Sabha can make laws on matters included in the State List.
(2) It controls the Executive by asking questions, supplementary questions, resolutions and
motions and through No-confidence Motion.
(3) It can introduce and enact Money and non-Money Bills.
(4) It can alone initiate the procedure for the removal of Vice President.
Which among the above is not a power of the Lok Sabha?
a. 1 and 2 only
b. 2, 3 and 4 only
c. 1, 2 and 3 only
d. 1 and 4 only

Answer: d
Explanation:
Under Article 249, Rajya Sabha gives the Union parliament power to make laws on matters
included in the State list. So statement 1 is not the power of Lok Sabha.
Similarly, Statement 4 is the power of the Rajya Sabha. As Vice President is Chairman
of Rajya Sabha, it is prerogative of Rajya Sabha to initiate the procedure for removal of Vice
President.
Statement 2 is the power of Lok Sabha as no-confidence motion is the exclusive concern
of Lok Sabha, and Rajya Sabha does not participate in it. Statement 3 is again a power
of Lok Sabha, whereby money bill can be introduced in Lok Sabha only as the people
directly elect its members. So answer option is d.

Q85. Consider the following:


(1) Unwritten Constitution
(2) Independent Judiciary
(3) Division of Powers
Which of the following is/are essential features of federalism?
a. 1 only
b. 2 and 3 only
c. 1 and 3 only
d. 1, 2 and 3

Answer: b
Explanation:
Statement 2 is correct - Division of Power is an essential feature of Federalism. There
has to be a written constitution to demarcate the division of powers through allocating
boundaries of jurisdiction. In India, the Seventh Schedule categorises the subjects
under Union List, State List and the Concurrent List. The Union and State have
exclusive jurisdiction on the Union and State List respectively whereas they both have
legislative power on subjects of concurrent list subject to centre having dominant powers in
case of a conflict. This is possible when there is a written documented constitution;
hence statement 1 is incorrect.
Statement 3 is correct. There has to be an independent judiciary to settle on disputes
between the centre and the states or among states whenever and wherever they arise. It
can be USA type where federal court enforces federal law and state courts implement state
laws or Indian type where it is integrated and judiciary enforces both. Thus, an independent
judiciary is an essential feature of federalism.

Q86. Consider the following statements:


(1) The Constitution declares India to be a federal country.
(2) The States are very much financially dependent on the Centre as per the constitutional
scheme of things.
(3) Though there is a Union Ministry of Agriculture, agriculture as a legislative subject comes
under the State List.
Which of the statements given above are correct?
a. 1 and 2 only
b. 2 and 3 only
c. 1 and 3 only
d. 1, 2 and 3

Answer: b
Explanation:
Statement 1 is incorrect. The Constitution does not explicitly mention the word ‘federalism’
anywhere. It instead states in Article 1 that India is a union of states. However, there is an
explicit federal tone in the Constitution. The Seventh Schedule contains the legislative
subjects of Union, States and also subjects where both have jurisdiction. There is proper
constitutional status given to states under Part VI so that they trace their power directly from
the constitution. The federal character is present so much so that the Supreme Court in
Bommai case held federalism to be the part of Basic Structure.
Statement 2 is correct. The Constitution, however, shows a unitary bias which is
prominently reflected in financial powers. The states cannot appropriate direct taxes which
is the major source of taxes. The states can borrow from the centre but external borrowings
are not allowed for states. The GST regime has taken away the state's power to tax sales
exclusively at their rates. States, thus, have to depend on proceeds of tax sharing
through finance commission as well as grant-in-aid from the centre.
Statement 3 is correct. Agriculture is a state list subject, though there is a Union Ministry
just to have uniform policy guidance on the sector where most are employed throughout the
country.

Q87. Which among the following statements is incorrect with regard to the National Human
Rights Commission (NHRC)?
(1) The Central government has established the National Human Rights Commission in
1993.
(2) The National Human Rights Commission (NHRC) does not include former Chief Justice
of a High Court.
(3) The Commission’s functions include undertaking and promoting research in the field of
human rights.
(4) The Commission has the power of prosecution and can initiate proceedings based on
the inquiry that it conducts.
Select the correct answer using the code given below:
a. 1 only
b. 2 and 3 only
c. 2 and 4 only
d. 2, 3 and 4 only

Answer: c
Explanation: The central government established the National Human Rights Commission
in 1993, to work as watchdogs against the violations of human rights. So statement 1 is
correct. NHRC is composed of a former chief justice of the Supreme Court of India, a former
judge of the Supreme Court, a former chief justice of a High Court and three other members
who have the knowledge and practical experience in matters relating to human rights. So
statement 2 is incorrect.
The Commission’s functions include inquiry at its initiative or on a petition presented to it by
a victim into a complaint of a violation of human rights; visit jails to study the condition of
the inmates; undertaking and promoting research in the field of human rights, etc. So
statement 3 is correct.
The Commission does not have the power of prosecution. It can merely make
recommendations to the government or recommend to the courts to initiate proceedings
based on the inquiry that it conducts. So statement 4 is incorrect. So right option is c

Q88. Consider the following statements:


(1) The reservation for women representatives exists at all the levels of the Panchayati Raj
system.
(2) There is no provision for reservations for the OBCs in Panchayats as per the
Constitution.
(3) The Scheduled Castes and Scheduled Tribes also have seats reserved for them in the
local governments, but the quantum of the reservation is not fixed as per the Constitution.
Which of the statements given above are correct?
a. 1 and 2 only
b. 2 and 3 only
c. 1 and 3 only
d. 1, 2 and 3

Answer: d
Explanation: All statements are correct. The Panchayati Raj system is a three-tier
structure consisting of Gram Panchayat at the village level, Anchalik Panchayat at the block
level and Zilla Parishad at the district level. As per the 73rd Amendment Act, there has to be
at least one-third of the total seats at each level reserved for women. Even among the
total seats reserved for the SCs and STs, at least one-third of the seats should be reserved
for women from the community. SCs and STs were given reservation according to their
population levels in the area. There is nothing fixed like that in government
employment or admissions to educational institutions.
OBC reservation in Panchayats have been a policy step from states and not provided in
the constitution as such. This is a step taken by each state based on their requirement
on the upliftment of the backward class.

Q89. Consider the following statements:


(1) Simple Majority of members present and voting in both the Houses of Parliament
(2) Special Majority of at least two-thirds of members present and voting in both the Houses
of Parliament
(3) Ratification by at least half of the State Legislatures.
(4) Majority of the total membership of each House of the Parliament.
Which of the following procedures of Constitutional Amendment has been adopted during
the introduction of GST?
a. 1, 2 and 3 only
b. 2, 3 and 4 only
c. 1, 3 and 4 only
d. 1, 2, 3 and 4

Answer: b
Explanation: Statement 2, 3 and 4 are correct.
The 101st Constitutional Amendment that led to the introduction of GST in India essentially
meant changing the federal distribution of taxation powers between centre and states. It is
an essential component of fiscal federalism. Thus, under Article 368 of the Constitution,
it was required to get the bill passed by a majority of at least two-third members
present and voting whose number should be the majority of the total membership of
house (hence statement 1 is incorrect). This should be carried out in both the houses
followed by ratification of at least half of the state legislatures. These procedures show
the rigid character of the Indian Constitution.

Q90. Consider the following statements about the 'basic structure' doctrine:
(1) The Supreme Court came out with this doctrine in the landmark Minerva Mills case
(2) The doctrine limits the constitutional amendment powers of the Parliament
(3) The contents that constitute the 'basic structure' had been clearly outlined by the
Supreme Court.
Which of the statements given above is/are incorrect?
a. 1 and 3 only
b. 2 and 3 only
c. 2 only
d. 1, 2 and 3

Answer: a
Explanation:
Statement 1 is incorrect. The 'basic structure' doctrine was declared by the Supreme
Court in the landmark Kesavananda Bharati case of 1973.
Statement 2 is correct. It allows the Parliament to amend any provision of the Constitution
to the limit that it does not change the basic structure of the Constitution. The basic
structure doctrine has balanced the rigidity as well as the flexible nature of the Constitution.
By saying that certain parts cannot be amended, it has underlined the rigid nature while by
allowing amendments to all others it has underlined the flexible nature of the amending
process.
Statement 3 is incorrect. The Supreme Court - the final interpreter of the Constitution -
has been declaring as to what constitutes the basic structure from time to time. In
various judgements, it has stated secularism, federalism, parliamentary democracy, the rule
of law etc. to be part of the basic structure.

Q91. Consider the following statements regarding a Constitutional Amendment Bill:


(1) These Bills can only be introduced in the Lok Sabha.
(2) It can be introduced only as a government bill.
(3) The prior recommendation of the President is not required.
(4) The President cannot withhold his assent or return the Bill for reconsideration of the
Parliament.
Which of the statements given above are correct?
a. 1 and 3 only
b. 3 and 4 only
c. 2, 3 and 4 only
d. 1, 2, 3 and 4

Answer: b
Explanation:
Statement 1 is incorrect. The Constitution Amendment Bills can be introduced in both
the Lok Sabha and Rajya Sabha. However, it cannot be introduced in the state
legislatures.
Statement 2 is incorrect. It can be introduced by a government minister or by a private
member of parliament as well.
Statement 3 and 4 are correct. There is no prior recommendation of President required
in the introduction of such bills. The amendment procedure is described in Article 368 and
is decided upon depending upon the content of the bill. After the passage, the only option
left with the President is of giving assent to the bill. He can neither withhold his assent
nor return the Bill for reconsideration of the Parliament. This shows that the elected
legislature is given more powers than the executive and that the President of India can
never achieve dictatorial designs by blocking constitutional amendments as happened in
many third world Presidential systems.

Q92. Consider the following statements about secularism in the Indian context:
(1) The Preamble to the Indian Constitution added the term secular through the 42nd
Constitutional Amendment of 1976
(2) It advocated a strict separation between religion and the State.
(3) Secularism is one of the basic structure of the Indian Constitution as defined by the
Supreme Court.
(4) There is no state religion of India.
Which of the statements given above are correct?
a. 1, 2 and 3 only
b. 1, 3 and 4 only
c. 2, 3 and 4 only
d. 1, 2, 3 and 4

Answer: b
Explanation: Statement 1 is correct. The word ‘secular’ was added to the preamble
through the 42nd Constitutional Amendment of 1976. It was the only time the Preamble was
amended. Along with it, the words ‘socialist’ as well as ‘integrity’ was added to the preamble
through the same amendment. Statement 2 is incorrect. Indian secularism is different
from western secularism. The latter talks about strict separation between religion and state
whereas in India secularism meant the state would take a principled distance from religion.
Statement 3 and 4 are correct. India would not have a state religion and that all the
religions shall be treated equally by the state. The state can intervene in religious issues
when it fails to reform itself with time and ensure dignity to all individuals. We can take the
example of state intervention in Triple Talaq and the Sabrimala issue to justify. The right
to freedom of religion has been granted as a fundamental right in the Indian constitution
and thus also made it justiciable so that fear of majority religious domination too is thwarted
away.

Q93. With regards to elections in India, consider the following statements:


(1) The Election Commission can adjudicate on an election dispute concerning the Lok
Sabha.
(2) The Chief Election Commissioner (CEC) presides over the Election Commission but
does not have more powers than the other Election Commissioners.
Which of the statements given above is/are correct?
a. 1 only
b. 2 only
c. Both 1 and 2
d. Neither 1 nor 2

Answer: b
Explanation: Election Commission does not have the authority to adjudicate on election
disputes about the Lok Sabha. The initial jurisdiction to deal with all doubts and disputes
relating to the elections to Parliament and State Legislatures vests in the High Court of the
State concerned, with a right of appeal to the Supreme Court (Article 329).
So statement 1 is incorrect.
The Chief Election Commissioner (CEC) presides over the Election Commission but does
not have more powers than the other Election Commissioners. The CEC and the two
Election Commissioners have equal powers to take all decisions relating to elections as a
collective body. So statement 2 is correct.
Q94. With regard to ensuring the independence of the Election Commission, consider the
following statements:
(1) The Chief Election Commissioner and the other Election Commissioners are appointed
by the President of India on the advice of a Committee consisting of the Prime Minister, the
Leader of Opposition and the Chief Justice of India.
(2) The Chief Election Commissioner and the Election Commissioners are appointed for a
five-year term or continue till the age of 65, whichever is earlier.
(3) The Chief Election Commissioner can be removed before the expiry of the term, by the
President if both Houses of the Parliament makes such a recommendation with a special
majority.
Which of the statements given above is/are incorrect?
a. 1 only
b. 2 only
c. 1 and 2 only
d. 2 and 3 only

Answer: c
Explanation: Chief Election Commissioner and other Election Commissioners are
appointed by the President of India on the advice of the Council of Ministers. It is, therefore,
possible for a ruling party to appoint a partisan person to the Commission who might favour
them in the elections. Many persons have suggested that a different method should be
followed that makes a consultation with the leader of the opposition and the Chief Justice
of India necessary for the appointment of CEC and Election Commissioners. So statement
1 is incorrect.
CEC and Election Commissioners are appointed for a six-year term or continue till the age
of 65, whichever is earlier. So statement 2 is also incorrect.
Article 324(6) states that the CEC can be removed from his office in like manner and on the
like grounds as a judge of the Supreme Court. The Constitution provides that a judge can
be removed only by an order of the President, based on a motion passed by both Houses
of Parliament. So, statement 3 is correct.
The procedure for removal of judges is
elaborated in the Judges Inquiry Act,
1968. The Act sets out the following
steps for removal from office:
Under the Act, an impeachment motion
may originate in either House of
Parliament. To initiate proceedings: (i) at
least 100 members of Lok Sabha may
give a signed notice to the Speaker, or
(ii) at least 50 members of Rajya Sabha
may give a signed notice to the
Chairman.
If the motion is admitted, the Speaker or
Chairman (who receives it) will constitute
a three-member committee to
investigate the complaint. It will
comprise: (i) a Supreme Court judge; (ii)
Chief Justice of a High Court; and (iii) a
distinguished jurist.
After concluding its investigation, the
Committee will submit its report to the
Speaker or Chairman, who will then lay
the report before the relevant House of
Parliament. If the report records a finding
of misbehaviour or incapacity, the motion
for removal will be taken up for
consideration and debated.
The motion for removal is required to be
adopted by each House of Parliament
by: (i) a majority of the total membership
of that House; and (ii) a majority of at
least two-thirds of the members of that
House present and voting. If the motion
is adopted by this majority, the motion
will be sent to the other House for
adoption.
Once the motion is adopted in both
Houses, it is sent to the President, who
will issue an order for the removal of the
judge.

Q95. With regard to the changes brought about by the 73rd Constitutional Amendment in
the Panchayati Raj institutions, consider the following statements:
(1) The three-tier structure was envisaged in the form of gram panchayat, mandal and zila
panchayat.
(2) All States have to mandatorily constitute all the three tiers irrespective of their size and
population.
(3) The role and functions of gram sabha would be decided by the Union Ministry of Rural
Development.
Which of the statements given above are incorrect?
a. 1 and 2 only
b. 2 and 3 only
c. 1 and 3 only
d. 1, 2 and 3

Answer: b
Explanation: In 1992, the 73rd and 74th constitutional amendments were passed by the
Parliament. The 73rd Amendment is about rural local governments (which are also known
as Panchayati Raj Institutions or PRIs) and the 74th amendment made the provisions
relating to urban local government (Nagarpalikas). The 73rd and 74th Amendments came
into force in 1993.
As per 73rd amendment, all States will now have a uniform three-tier Panchayati Raj
structure. At the base is the ‘Gram Panchayat‘. A Gram Panchayat covers a village or group
of villages. The intermediary level is the Mandal (also referred to as Block or Taluka). These
bodies are called Mandal or Taluka Panchayats. The intermediary level body need not be
constituted in smaller States (with a population less than 20 lakhs). At the apex is the Zila
Panchayat covering the entire rural area of the District.
The Gram Sabha would comprise of all the adult members registered as voters in the
Panchayat area. Its role and functions are decided by State legislation. Therefore statement
3 is incorrect.

Q96. Consider the following statements with regard to the Indian Vice President:
(1) There is no difference in tenure and election method of the Vice President and the
President of India.
(2) The Vice President acts as the ex-officio Chairman of the Rajya Sabha.
The Vice President may be removed from his office by a resolution of the Lok Sabha passed
by a majority and agreed to by the Rajya Sabha.
(3) B. D. Jatti acted as the President on the death of Fakhruddin Ali Ahmed until a new
President was elected.
Which of the statements given above is/are incorrect?
a. 1 and 2 only
b. 2 only
c. 1 and 3 only
d. 2 and 4 only
Answer: c
Explanation: The Vice President is elected for five years. His election method is similar to
that of the President, the only difference is that members of State legislatures are not part
of the electoral college. Therefore statement 1 is incorrect.
The Vice President may be removed from his office by a resolution of the Rajya Sabha
passed by a majority and agreed to by the Lok Sabha. So statement 3 is incorrect.
The Vice President acts as the ex-officio Chairman of the Rajya Sabha and takes over the
office of the President when there is a vacancy by reasons of death, resignation, removal
by impeachment or otherwise. The Vice President acts as the President only until a new
President is elected. Statement 2 is correct.
B. D. Jatti acted as President on the death of Fakhruddin Ali Ahmed until a new President
was elected. Statement 4 is correct.

Q97. Which of the following statements given below is incorrect?


a. The President can send back the advice given by the Union Council of Ministers for
reconsideration.
b. Usually, in the parliamentary system, a leader who has the support of the majority in the
Lok Sabha would be appointed as the Prime Minister.
c. The President has veto power by which he can withhold or refuse to give assent to all
Bills passed by the Parliament.
d. After an election, if no leader has a clear majority in the Lok Sabha, the President has to
decide whom to appoint as the Prime Minister.

Answer: c
Explanation: There are at least three situations where the President can exercise the
powers using his or her discretion. In the first place, the President can send back the advice
given by the Council of Ministers and ask the Council to reconsider the decision. In doing
this, the President acts on his (or her) own discretion. So statement a is correct.
Secondly, the President also has veto power by which he can withhold or refuse to give
assent to Bills (other than Money Bill) passed by the Parliament. Every bill passed by the
Parliament goes to the President for his assent before it becomes a law. The President
cannot send the Money bills back to the Parliament asking it to reconsider the bill. Thus
Statement c is incorrect.
Formally, the President appoints the Prime Minister. Normally, in the parliamentary system,
a leader who has the support of the majority in the Lok Sabha would be appointed as Prime
Minister and the question of discretion would not arise. Statement b is correct.
But imagine a situation when after an election, no leader has a clear majority in the Lok
Sabha. In such a situation, the President has to use his own discretion in judging who really
may have the support of the majority or who can actually form and run the government.
Statement d is correct.

Q98. Which among the following is/are disadvantages of bicameral legislatures?


(1) It is preferred by countries with large size and much diversity as it gives representation
to all sections and all geographical regions of the country.
(2) A bicameral legislature makes it possible to have every decision reconsidered, and
prevents hasty decisions.
Select the correct answer using the code given below:
a. 1 only
b. 2 only
c. Both 1 and 2
d. Neither 1 nor 2

Answer: d
Explanation: Countries with large size and much diversity usually prefer to have two
houses of the national legislature to give representation to all sections in the society and to
give representation to all geographical regions or parts of the country. So statement 1 is
an advantage, not disadvantage so incorrect.
In a bicameral legislature, every decision taken by one house goes to the other house for
its decision. This means that every bill and policy would be discussed twice. This ensures
a double check on every matter. Even if one house takes a decision in haste, that decision
will come for discussion in the other house and reconsideration will be possible. So
statement 2 is also an advantage and not disadvantage, therefore incorrect.

Q99. Consider the following statements with regard to Executive:


(1) In a Presidential system, the President is the Head of State as well as the Head of
Government, thus the office of President is very powerful, both in theory and practice.
(2) Canada, like the USA, has a Presidential system and executive powers are in the hands
of the President.
Which of the statements given above is/are correct?
a. 1 only
b. 2 only
c. Both 1 and 2
d. Neither 1 nor 2

Answer: a
Explanation: In a presidential system, the president is the head of state as well as head of
government. In this system, the office of president is very powerful, both in theory and
practice. Countries with such a system include the United States, Brazil and most nations
in Latin America. So, statement 1 is correct.
Canada has a parliamentary democracy with a constitutional monarchy where Queen
Elizabeth II is the formal chief of state and the prime minister is the head of government.
So, statement 2 is incorrect.

Q100. With regard to the Objectives Resolutions, which of the statements given below is
incorrect?
a. It is the best summary of the principles that the nationalist movement brought to the
Constituent Assembly as the Resolution defined the aims of the Assembly.
b. This Resolution was moved by Jawahar Lal Nehru on 15th August 1947.
c. This Resolution resolved to promote world peace and the welfare of humanity.
d. Based on this Resolution, the Constitution gave institutional expression to commitments
of equality, liberty, democracy, sovereignty and a cosmopolitan identity

Answer: b
Explanation: Objectives resolution was moved by Jawahar Lal Nehru in 1946 and adopted
by the assembly in January 1947. So statement b is incorrect.
Perhaps the best summary of the principles that the nationalist movement brought to the
Constituent Assembly is the Objectives Resolution (the resolution that defined the aims of
the Assembly). This resolution encapsulated the aspirations and values behind the
Constitution. Based on this resolution, our Constitution gave institutional expression to
these fundamental commitments: equality, liberty, democracy, sovereignty and a
cosmopolitan identity
According to the Objectives Resolution, India is an independent, sovereign, and a republic.
All powers and authority of sovereign and independent India and its constitution shall flow
from the people. The land would make a full and willing contribution to the promotion of
world peace and welfare of humanity. So, the rest of the statements a, c and d are correct.

S-ar putea să vă placă și